You are on page 1of 106

Eighth Edition

GATE
ELECTRONICS & COMMUNICATION

Digital Electronics
Vol 6 of 10

R. K. Kanodia
Ashish Murolia

NODIA & COMPANY

GATE Electronics & Communication Vol 6, 8e


Digital Electronics
RK Kanodia & Ashish Murolia

Copyright By NODIA & COMPANY

Information contained in this book has been obtained by author, from sources believes to be reliable. However,
neither NODIA & COMPANY nor its author guarantee the accuracy or completeness of any information herein,
and NODIA & COMPANY nor its author shall be responsible for any error, omissions, or damages arising out
of use of this information. This book is published with the understanding that NODIA & COMPANY and its
author are supplying information but are not attempting to render engineering or other professional services.

MRP 490.00

NODIA & COMPANY

B 8, Dhanshree Ist, Central Spine, Vidyadhar Nagar, Jaipur 302039


Ph : +91 141 2101150,
www.nodia.co.in
email : enquiry@nodia.co.in
Printed by Nodia and Company, Jaipur

To Our Parents

Preface to the Series


For almost a decade, we have been receiving tremendous responses from GATE aspirants for our earlier books:
GATE Multiple Choice Questions, GATE Guide, and the GATE Cloud series. Our first book, GATE Multiple
Choice Questions (MCQ), was a compilation of objective questions and solutions for all subjects of GATE
Electronics & Communication Engineering in one book. The idea behind the book was that Gate aspirants
who had just completed or about to finish their last semester to achieve his or her B.E/B.Tech need only to
practice answering questions to crack GATE. The solutions in the book were presented in such a manner that
a student needs to know fundamental concepts to understand them. We assumed that students have learned
enough of the fundamentals by his or her graduation. The book was a great success, but still there were a large
ratio of aspirants who needed more preparatory materials beyond just problems and solutions. This large ratio
mainly included average students.
Later, we perceived that many aspirants couldnt develop a good problem solving approach in their B.E/B.
Tech. Some of them lacked the fundamentals of a subject and had difficulty understanding simple solutions.
Now, we have an idea to enhance our content and present two separate books for each subject: one for theory,
which contains brief theory, problem solving methods, fundamental concepts, and points-to-remember. The
second book is about problems, including a vast collection of problems with descriptive and step-by-step
solutions that can be understood by an average student. This was the origin of GATE Guide (the theory book)
and GATE Cloud (the problem bank) series: two books for each subject. GATE Guide and GATE Cloud were
published in three subjects only.
Thereafter we received an immense number of emails from our readers looking for a complete study package
for all subjects and a book that combines both GATE Guide and GATE Cloud. This encouraged us to present
GATE Study Package (a set of 10 books: one for each subject) for GATE Electronic and Communication
Engineering. Each book in this package is adequate for the purpose of qualifying GATE for an average student.
Each book contains brief theory, fundamental concepts, problem solving methodology, summary of formulae,
and a solved question bank. The question bank has three exercises for each chapter: 1) Theoretical MCQs,
2) Numerical MCQs, and 3) Numerical Type Questions (based on the new GATE pattern). Solutions are
presented in a descriptive and step-by-step manner, which are easy to understand for all aspirants.
We believe that each book of GATE Study Package helps a student learn fundamental concepts and develop
problem solving skills for a subject, which are key essentials to crack GATE. Although we have put a vigorous
effort in preparing this book, some errors may have crept in. We shall appreciate and greatly acknowledge
all constructive comments, criticisms, and suggestions from the users of this book. You may write to us at
rajkumar.kanodia@gmail.com and ashish.murolia@gmail.com.

Acknowledgements
We would like to express our sincere thanks to all the co-authors, editors, and reviewers for their efforts in
making this project successful. We would also like to thank Team NODIA for providing professional support
for this project through all phases of its development. At last, we express our gratitude to God and our Family
for providing moral support and motivation.
We wish you good luck !
R. K. Kanodia
Ashish Murolia

SYLLABUS
GATE Electronics & Communications
Boolean algebra, minimization of Boolean functions; logic gates; digital IC families (DTL,
TTL, ECL, MOS, CMOS). Combinatorial circuits: arithmetic circuits, code converters,
multiplexers, decoders, PROMs and PLAs. Sequential circuits : latches and flip-flops,
counters and shift-registers. Sample and hold circuits, ADCs, DACs. Semiconductor memories.
Microprocessor(8085): architecture, programming, memory and I/O interfacing.
IES Electronics & Telecommunication
Transistor as a switching element ; Simplification of Boolean functions, Karnaguh map
, Boolean algebra, and applications; IC logic families : DTL, ECL, TTL, NMOS, CMOS
and PMOS gates and their comparison; Full adder , Half adder; IC Logic gates and their
characteristics; Digital comparator; Multiplexer Demultiplexer; Flip flops. J-K, R-S, T and
D flip-flops; Combinational logic Circuits; Different types of registers and counters Waveform
generators. Semiconductor memories.A/D and D/A converters. ROM an their applications.
**********

CONTENTS
CHAP 1

NUMBER SYSTEM AND CODES

1.1

INTRODUCTION

1.2

ANALOG AND DIGITAL SYSTEMS

1.2.1
1.2.2

2
2

1.3

Decimal Number System


2
Binary Number System
3
Octal Number System
3
Hexadecimal Number System

NUMBER SYSTEM CONVERSION

1.4.1
1.4.2
1.4.3
1.4.4
1.4.5
1.4.6
1.4.7
1.4.8
1.5

Advantages of Digital System


Limitations of Digital System

NUMBER SYSTEMS

1.3.1
1.3.2
1.3.3
1.3.4
1.4

Decimal-to-Binary Conversion
5
Decimal-to-Octal Conversion
6
Decimal-to-Hexadecimal Conversion
7
Octal-to-Binary conversion 7
Binary-to-Octal Conversion 7
Hexadecimal-to-Binary Conversion 8
Binary-to-Hexadecimal Conversion 8
Hexadecimal-to-Octal and Octal-to-Hexadecimal Conversion 8

BASIC BINARY ARITHMETIC

1.5.1
1.5.2
1.5.3
1.5.4

9
9

Binary
Binary
Binary
Binary

Addition
9
Subtraction
Multiplication
Division
9

1.6

COMPLEMENTS OF NUMBERS

1.7

NUMBER REPRESENTATION IN BINARY

1.7.1
1.7.2
1.7.3
1.8

1.9

10

Sign-Magnitude Representation
1s Complement Representation
2s Complement Representation

11

11
11
12

COMPLEMENT BINARY ARITHMETIC

13

1.8.1
1.8.2
1.8.3
1.8.4

13
13
14
15

Addition Using 1s Complement


Subtraction Using 1s Complement
Addition Using 2s Complement
Subtraction using 2s Complement

HEXADECIMAL ARITHMETIC

1.9.1
1.9.2
1.10

15

Hexadecimal Arithmetic Using 1s or 2s Complements


Hexadecimal Subtraction Using 15s or 16s Complement

OCTAL ARITHMETIC

16

1.10.1 Octal Arithmetic using 1s or 2s Complements


1.10.2 Octal Subtraction using 7s or 8s complement

16
16

15
15

1.11

DECIMAL ARITHMETIC

17

1.11.1 Decimal Arithmetic Using 1s or 2s Complements


17
1.11.2 Decimal Subtraction Using 9s and 10s Complement
1.12

BINARY CODES

1.13

BINARY CODED DECIMAL (BCD) CODE OR 8421 CODE

17

18
20

1.13.1 BCD-to-Binary Conversion 20


1.13.2 Binary-to-BCD Conversion 20
1.14

BCD ARITHMETIC 20

1.14.1 BCD Addition


1.14.2 BCD Subtraction

21
21

1.15

THE EXCESS-3 CODE

22

1.16

GRAY CODE

23

1.16.1 Binary-to-Gray Code Conversion


1.16.2 Gray-to-Binary Code Conversion
1.16.3 Applications of Gray Code
EXERCISE 1.1

25

EXERCISE 1.2

31

EXERCISE 1.3

33

SOLUTIONS 1.1

41

SOLUTIONS 1.2

53

SOLUTIONS 1.3

58

23
24
24

CHAPTER 2 BOOLEAN ALGEBRA AND LOGIC SIMPLIFICATION


2.1

INTRODUCTION

2.2

BOOLEAN ALGEBRA

63

2.2.1
2.2.2

63
64

2.3

BASIC BOOLEAN OPERATIONS

2.3.1
2.3.2
2.3.3
2.4

2.5

Logic Levels
Truth Table

63

64

Boolean Addition (Logical OR)


Boolean Multiplication (Logical AND)
Logical NOT
65

THEOREMS OF BOOLEAN ALGEBRA

66

2.4.1
2.4.2
2.4.3
2.4.4
2.4.5
2.4.6
2.4.7
2.4.8
2.4.9
2.4.10
2.4.11
2.4.12
2.4.13

68

Complementation Laws
66
AND Laws
66
OR Laws 66
Commutative Laws
67
Associative Laws
67
Distributive Law
67
Redundant Literal Rule
67
Idempotent Law
67
Absorption Law
67
Consensus Theorem
67
Transposition Theorem
68
De Morgans Theorem
68
Shannons Expansion Theorem

64
65

SIMPLIFICATION OF BOOLEAN EXPRESSIONS USING BOOLEAN ALGEBRA

2.5.1

Complement of Boolean Function

69

68

2.5.2
2.5.3
2.6

LOGIC GATES

2.6.1
2.6.2
2.7

Principal of Duality
69
Relation Between Complement and Dual
69

Logic Levels
70
Types of Logic Gates

UNIVERSAL GATE

2.7.1
2.7.2

70

75

NAND Gate as a Universal Gate


NOR Gate as a Universal Gate

75
77

2.8

ALTERNATE LOGIC-GATE REPRESENTATIONS

2.9

BOOLEAN ANALYSIS OF LOGIC CIRCUITS

2.9.1
2.9.2
2.10

69

79
80

Converting Boolean Expressions to Logic Diagram


Converting Logic to Boolean Expressions
81

80

CONVERTING LOGIC DIAGRAMS TO NAND / NOR LOGIC

2.10.1 NAND-NAND Logic


2.10.2 NOR-NOR Logic
83
EXERCISE 2.1

84

EXERCISE 2.2

105

EXERCISE 2.3

107

SOLUTIONS 2.1

117

SOLUTIONS 2.2

144

SOLUTIONS 2.3

148

82

82

CHAPTER 3 THE K-MAP


3.1

INTRODUCTION

3.2

REPRESENTATION FOR BOOLEAN FUNCTIONS

3.2.1
3.2.2
3.3

Sum-of-Products (SOP)
Product-of-Sum (POS)

155

156
156

STANDARD OR CANONICAL SUM-OF-PRODUCTS (SOP) FORM 156

3.3.1
3.3.2
3.3.3
3.4

155

Minterm
157
157
S Notation
Converting SOP Form to Standard SOP Form

158

STANDARD OR CANONICAL PRODUCT-OF-SUMS (POS) FORM 158

3.4.1
3.4.2
3.4.3

Maxterm 158
159
P Notation
Converting POS Form to standard POS Form

159

3.5

CONVERTING STANDARD SOP FORM TO STANDARD POS FORM

3.6

BOOLEAN EXPRESSIONS AND TRUTH TABLES

3.7

CALCULATION OF TOTAL GATE INPUTS USING SOP AND POS

3.8

KARNAUGH MAP (K-MAP)

3.8.1
3.8.2
3.8.3
3.9

PLOTTING A K-MAP

3.9.1
3.9.2
3.9.3

162

Structure of K-map
163
Another Structure of K-map
Cell Adjacency
165

165

166

Plotting Standard SOP on K-map


Plotting Standard POS on K-map
Plotting a Truth Table on K-map

166
166
166

160

161
162

3.10

GROUPING OF CELLS FOR SIMPLIFICATION

3.10.1
3.10.2
3.10.3
3.10.4

Grouping of Two adjacent Cells (Pair)


Grouping of Four Adjacent Cells (Quad)
Grouping of Eight Adjacent Cells (Octet)
Redundant Group
169

3.11

MINIMIZATION OF SOP EXPRESSIONS

169

3.12

MINIMIZATION OF POS EXPRESSIONS

170

3.13

CONVERTING SOP TO POS AND VICE-VERSA

3.14

DONT CARE CONDITIONS

166

166
167
168

170

171

3.14.1 K-map Simplification With Dont Care Conditions 171


3.14.2 Conversion of Standard SOP to Standard POS with Dont Care Conditions
3.15

K-MAPS FOR MULTI-OUTPUT FUNCTIONS

3.16

LIMITATIONS OF K-MAP 172

EXERCISE 3.1

173

EXERCISE 3.2

186

EXERCISE 3.3

188

SOLUTIONS 3.1

192

SOLUTIONS 3.2

223

SOLUTIONS 3.3

228

171

171

CHAPTER 4 COMBINATIONAL CIRCUITS


4.1

INTRODUCTION

4.2

DESIGN PROCEDURE FOR COMBINATION LOGIC CIRCUITS

4.3

ADDERS

4.3.1
4.3.2
4.4

232
233
235

Half-Subtractor
Full-Subtractor

235
236

4.5

BINARY PARALLEL ADDER

237

4.6

CARRY LOOK-AHEAD ADDER

238

4.6.1
4.6.2
4.6.3

238
239
239

Carry Generation
Carry Propagation
Look Ahead Expressions

4.7

SERIAL ADDER

240

4.8

COMPARATOR

241

4.8.1
4.8.2
4.9

4.10

1-bit Magnitude Comparator


2-bit Magnitude Comparator

MULTIPLEXER

4.9.1
4.9.2
4.9.3
4.9.4

231

232

Half-Adder
Full-Adder

SUBTRACTORS

4.4.1
4.4.2

231

241
242

244

2-to-1 Multiplexer
245
4-to-1 Multiplexer
245
Implementation of Higher Order Multiplexers using Lower Order Multiplexers
Applications of Multiplexers
247

DEMULTIPLEXER

4.10.1 1-to-2 Demultiplexer


4.10.2 1-to-8 Demultiplexer

247

248
249

247

4.10.3 Applications of Demultiplexers


250
4.10.4 Comparison between Multiplexer and Demultiplexer
4.11

DECODER 251

4.11.1 2-to-4 Line Decoder


4.11.2 Applications of Decoder
4.12

250

ENCODERS

253

4.12.1 Octal-to-Binary Encoder


4.12.2 Decimal-to-BCD Encoder
4.13

PRIORITY ENCODERS

256

4.14

CODE CONVERTERS

257

4.15

PARITY GENERATOR

259

4.15.1 Even Parity Generator


4.15.2 Odd Parity Generator
EXERCISE 4.1

262

EXERCISE 4.2

281

EXERCISE 4.3

284

SOLUTIONS 4.1

291

SOLUTIONS 4.2

314

SOLUTIONS 4.3

318

252
253
253
254

260
260

CHAPTER 5 SEQUENTIAL CIRCUITS


5.1

INTRODUCTION

5.2

SEQUENTIAL LOGIC CIRCUITS

323

5.3

LATCHES AND FLIP-FLOPS

324

5.3.1
5.3.2
5.4

S - R Latch using NOR Gates


S - R Latch using NAND Gates

FLIP-FLOPS

5.5.1
5.5.2
5.5.3
5.5.4
5.6

General Block Diagram of a Latch or Flip-flop


Difference between Latches and Flip-flops
325

S-R LATCH 325

5.4.1
5.4.2
5.5

323

327

S-R Flip-Flop
D-Flip Flop
J-K Flip-Flop
T Flip-Flop

327
328
329
331

TRIGGERING OF FLIP-FLOPS

5.6.1
5.6.2
5.6.3
5.6.4
5.6.5
5.6.6

325
326

332

Level Triggering
332
Edge Triggering
332
Edge Triggered S - R Flip Flop
Edge Triggered D Flip-Flop
Edge Triggered J - K Flip-Flop
Edge Triggered T -Flip-Flop

334
336
337
339

5.7

OPERATING CHARACTERISTIC OF FLIP-FLOPS 340

5.8

APPLICATION OF FLIP-FLOPS

5.9

REGISTER

5.9.1
5.9.2

343

Buffer Register
Shift Register

343
344

342

324

5.9.3
5.10

Applications of Shift Registers

345

COUNTER 345

5.10.1 Asynchronous and Synchronous Counter


5.10.2 Up-Counter and Down-Counter
5.10.3 MOD Number or Modulus of a Counter
5.11

SHIFT REGISTER COUNTERS

348

5.11.1 Ring Counter


5.11.2 Johnson Counter

349

EXERCISE 5.1

352

EXERCISE 5.2

369

EXERCISE 5.3

372

SOLUTIONS 5.1

383

SOLUTIONS 5.2

402

SOLUTIONS 5.3

407

345
346
348

348

CHAPTER 6 LOGIC FAMILIES


6.1

INTRODUCTION

6.2

CLASSIFICATION OF DIGITAL LOGIC FAMILY

6.3

CHARACTERISTIC PARAMETERS OF DIGITAL LOGIC FAMILY

6.3.1
6.3.2
6.3.3
6.3.4
6.3.5
6.3.6
6.3.7
6.3.8
6.3.9
6.4

Speed of Operation
414
Power Dissipation
415
Voltage Parameters
415
Current Parameters
416
Noise Immunity or Noise Margin
Fan-In
417
Fan-out
417
Operating Temperature
417
Speed Power Product
417

RESISTOR-TRANSISTOR LOGIC (RTL)

6.4.1
6.4.2
6.5

413

Circuit Operation
Drawbacks of RTL Family

413

416

418

418
418

DIRECT COUPLED TRANSISTOR LOGIC (DCTL) 419

6.5.1

Circuit Operation

419

6.6

DIODE TRANSISTOR LOGIC (DTL)

6.7

TRANSISTOR-TRANSISTOR LOGIC (TTL)

6.8

TTL CIRCUIT OUTPUT CONNECTION

6.8.1
6.8.2
6.8.3

Totem-pole Output
Open-collector Output
Tri-state Output
423

6.9

TTL SUBFAMILIES

6.10

EMITTER COUPLED LOGIC (ECL)

419
421
422

422
423

424
425

6.10.1 ECL OR/NOR Gate


426
6.10.2 ECL Characteristics
427
6.10.3 Advantages and Disadvantages of ECL Family
6.11

INTEGRATED INJECTION LOGIC (I L)


2

6.11.1 Characteristic of I2L

428

428

427

414

6.11.2
6.11.3
6.11.4
6.11.5
6.11.6
6.12

NMOS Inverter
430
NMOS NAND Gate
431
NMOS NOR Gate
432
Characteristics of MOS Logic

433

COMPLEMENTARY METAL OXIDE SEMICONDUCTOR (CMOS) LOGIC

6.13.1
6.13.2
6.13.3
6.13.4
6.13.5
6.14

429
429

METAL OXIDE SEMICONDUCTOR (MOS) LOGIC 430

6.12.1
6.12.2
6.12.3
6.12.4
6.13

I2L Inverter
428
428
I2L NAND Gate
I2L NOR Gate
429
Advantages of I2L
Disadvantages of I2L

CMOS Inverter
434
CMOS NAND Gate
434
CMOS NOR Gate
435
Characteristics of CMOS Logic
436
Advantages and Disadvantages of CMOS Logic

COMPARISON OF VARIOUS LOGIC FAMILIES

EXERCISE 6.1

439

EXERCISE 6.2

455

EXERCISE 6.3

458

SOLUTIONS 6.1

465

SOLUTIONS 6.2

485

SOLUTIONS 6.3

490

437

437

CHAPTER 7 INTERFACING TO ANALOG


7.1

INTRODUCTION

7.2

DIGITAL TO ANALOG CONVERTER

7.2.1
7.3

7.4

7.5

496

R - 2R Ladder Type DAC


Weighted Resistor Type DAC

496
497
497

7.4.1
7.4.2
7.4.3

499

Sample-and-hold circuit
498
Quantization and Encoding
Parameters of ADC
499

ADC CIRCUITS

500

Flash Type A/D Converter 500


Counting A/D Converter
501
Dual Slope Type A/D Converter
503
Successive Approximation Type ADC

ASTABLE MULTIVIBRATOR

7.6.1
7.6.2
7.6.3
7.7

495

496

ANALOG-TO-DIGITAL CONVERTER

7.5.1
7.5.2
7.5.3
7.5.4
7.6

Parameters of DAC

DAC CIRCUITS

7.3.1
7.3.2

495

503

504

Astable Multivibrator Using BJT


505
Astable Multivibrator Using 555 Timer
Astable Multivibrator Using Op-amps

507
507

MONOSTABLE MULTIVIBRATOR 508

7.7.1
7.7.2

Monostable Multivibrator Using BJT


Monostable Multivibrator Using 555 Timer

508
510

433

7.8

SCHMITT TRIGGER

7.8.1
7.8.2

511

Schmitt Trigger Using BJT


Schmitt Trigger Using 555 Timer

EXERCISE 7.1

515

EXERCISE 7.2

532

EXERCISE 7.3

535

SOLUTIONS 7.1

541

SOLUTIONS 7.2

564

SOLUTIONS 7.3

568

512
513

CHAPTER 8 MICROPROCESSOR
8.1

INTRODUCTION

8.2

MICROCOMPUTER

8.2.1
8.2.2
8.2.3
8.3

8.6

572

FETCH
573
EXECUTE

572

573

MICROPROCESSOR ARCHITECTURE

8.4.1
8.4.2
8.4.3
8.4.4
8.4.5
8.4.6
8.5

571

Memory
572
Input-Output Interfacing
System Bus
572

MICROPROCESSOR OPERATION

8.3.1
8.3.2
8.4

571

System Bus
573
Arithmetic Logic Unit (ALU)
Registers
574
Program Counter (PC)
574
Flags
574
Timing and Control Unit

573

573

574

PIN DIAGRAM OF 8085 MICROPROCESSOR

574

8.5.1
8.5.2
8.5.3
8.5.4
8.5.5

576

Address and Data Bus


575
Control and Status Signals
575
Power Supply and Clock Frequency
Interrupts and Other Operations
576
Serial I/O Ports
577

INSTRUCTION SET

8.6.1
8.6.2
8.6.3
8.6.4
8.6.5

577

Data Transfer Instructions


Arithmetic Instructions
Branching Instructions
Logic Instructions
Control Instructions

EXERCISE 8.1

589

EXERCISE 8.2

602

EXERCISE 8.3

605

SOLUTIONS 8.1

609

SOLUTIONS 8.2

621

SOLUTIONS 8.3

625

577
579
581
584
587

***********

GATE STUDY PACKAGE

Electronics & Communication

Sample Chapter of

Digital Electronics

(Vol-6, GATE Study Package)

Page 63
Chap 2

CHAPTER 2

Boolean Algebra and


Logic Simplification

BOOLEAN ALGEBRA AND LOGIC SIMPLIFICATION

2.1

i. n

INTRODUCTION

o
c
.
a

This chapter, concerned with the basic study of Boolean algebra and
simplification theory, includes the following topics:

Introduction to Boolean algebra: logic levels, truth table.

Basic Boolean operations: addition, multiplication, not operation

Various theorems of Boolean algebra

Meaning of positive and negative logic

Various types of logic gates: AND, OR, NOT, NAND, NOR, XOR,
XNOR gates.

.
w
w

2.2

o
n

i
d

Universal logic gates; conversion of logic diagrams to universal


logic gates.

Boolean analysis of logic circuits.

BOOLEAN ALGEBRA

Boolean algebra is mathematics of logic. It is one of the most basic


tools which is used in the analysis and synthesis of logic circuit. In
Boolean algebra, often the variables are represented by capital letters
such as A, B , C , X , Y , Z . The Boolean value of a variable is either
logic 0 or logic 1. These, 0 and 1, are known as Boolean constants.

2.2.1

Logic Levels
Boolean logic variable 0 or 1 is not used to represent actual numbers
but it is used to represent the state of voltage variable called logical
level. Commonly used representation of logic levels are shown in Table
below.
Table 2.1: Representation of Logic Levels for Boolean Variables

Logic 0

Logic 1

False

True

Open switch

Close switch

Buy Online: shop.nodia.co.in


*Shipping Free*

*Maximum Discount*

GATE STUDY PACKAGE


Electronics & Communication
10 Subject-wise books by R. K. Kanodia
General Aptitude
Engineering Mathematics
Networks
Electronic Devices
Analog Electronics
Digital Electronics
Signals & Systems
Control Systems Communication Systems
Electromagnetics

Page 64
Chap 2
Boolean Algebra and
Logic Simplification

2.2.2

Logic 0

Logic 1

Low

High

No

Yes

OFF

ON

Truth Table
A truth table represents the relation between all inputs and possible
outputs of any logic device or logic circuit in a tabular form. The
number of inputs may vary from one to many depending upon the
device or complexity of the circuit. Number of output also varies in this
way and may be one or more. For different digital circuits, some of the
examples of truth table are given below.

in
.
o

Table 2.2: Examples of Truth Tables for 1-input, 2-input and 3-input
Circuits

c
.
ia

d
o

n
.
w
w

w
2.3

BASIC BOOLEAN OPERATIONS

Boolean algebra uses only three basic operations, namely


1. OR operation

2.3.1

2.

AND operation

3.

NOT operation

Boolean Addition (Logical OR)


The OR operation in Boolean algebra is similar to addition in ordinary
algebra i.e., OR means logical addition operation. The logical OR
operation on A and B is denoted by

Buy Online: shop.nodia.co.in


*Shipping Free*

*Maximum Discount*

GATE STUDY PACKAGE

Electronics & Communication

Sample Chapter of

Digital Electronics

(Vol-6, GATE Study Package)

where + is the OR operator


Y = A + B,
The output Y corresponding to various combinations of inputs, A
and B , is shown in Table 2.3 below.

Page 65
Chap 2
Boolean Algebra and
Logic Simplification

Table 2.3: Truth Table for OR Operation

Input

Output

Y = A+B

i. n

o
c
.
a

NOTE :

The minimum number of inputs for OR operation is two. The number of outputs
is always one, irrespective of the number of inputs.

2.3.2

i
d

Boolean Multiplication (Logical AND)

o
n

The AND operation in Boolean algebra is similar to multiplication in


ordinary algebra i.e, AND performs logical multiplication operation.
Let A and B be two Boolean variables. Then, the logical AND
operation on A and B is denoted by
Y = A : B,
where : is the AND operator. The output Y corresponding to various
combinations of inputs, A and B , is shown in Table 2.4 below.

.
w
w

Table 2.4: Truth table for AND operation

Input

Output

Y = AB

NOTE :

The minimum number of inputs for AND operation is two. The number of output
is always one, irrespective of the number of inputs.

2.3.3

Logical NOT
NOT is the simplest of the three basic operations of Boolean algebra.
It is also known as inversion and complement. The NOT operation is
indicated by a bar - over the variable. If A is a variable, then NOT
of A is expressed as A . The truth Table of the NOT operation is shown
in Table 2.5.

Buy Online: shop.nodia.co.in


*Shipping Free*

*Maximum Discount*

GATE STUDY PACKAGE


Electronics & Communication
10 Subject-wise books by R. K. Kanodia
General Aptitude
Engineering Mathematics
Networks
Electronic Devices
Analog Electronics
Digital Electronics
Signals & Systems
Control Systems Communication Systems
Electromagnetics

Table 2.5: Truth table for NOT operation

Page 66
Chap 2
Boolean Algebra and
Logic Simplification

Input

Output

Y=A

NOTE :

Logical NOT is the only Boolean operation which must be performed with only
one operand or one input. Note that in some texts, the NOT operation is also
presented as Al .

2.4

THEOREMS OF BOOLEAN ALGEBRA

The theorems of Boolean algebra can be used to simplify many complex


Boolean expression and also to transform the given expression into a
more useful and meaningful equivalent expression. These theorems are
discussed as below.

2.4.1

in
.
o

c
.
ia

Complementation Laws

The term complement implies to invert, i.e. to change 1s to 0s and 0s


to 1s. The five laws of complementation are as follows:
1. The complement of 0 is 1, i.e. 0 = 1

d
o

n
.
w
w

The complement of 1 is 0, i.e. 1 = 0

2.

If A = 0 , then A = 1

3.

If A = 1, then A = 0

4.
5.

2.4.2

The double complementation does not change the function, i.e.


A=A

AND Laws
The four AND laws are as follows:
1. Null Law: A : 0 = 0

2.4.3

2.

Identity Law: A : 1 = A

3.

A:A = A

4.

A:A = 0

OR Laws
The four OR laws are as follows:
1. Null Law: A + 0 = A
2.

Identity Law: A + 1 = 1

3.

A+A = A

4.

A+A = 1

Buy Online: shop.nodia.co.in


*Shipping Free*

*Maximum Discount*

GATE STUDY PACKAGE

Electronics & Communication

Digital Electronics

Sample Chapter of

2.4.4

(Vol-6, GATE Study Package)

Commutative Laws
Commutative law states that the order of the variable in OR and AND
operations is not important. The two commutative laws are

Page 67
Chap 2
Boolean Algebra and
Logic Simplification

A+B = B+A
A:B = B:A

2.4.5

Associative Laws
Associative law states that the grouping of variables in AND or OR
expression does not affect the result. There are two associative laws.
A + _B + C i = _A + B i + C
A : _B : C i = _A : B i : C

2.4.6

i. n

o
c
.
a

Distributive Law

The distributive laws allow factoring or multiplying out of expressions.


There are two distributive laws

i
d

A _B + C i = AB + AC
A + BC = _A + B i_A + C i

2.4.7

o
n

Redundant Literal Rule

.
w
w

This law states that ORing of a variable with the AND of the
complement of that variable with another variable, is equal to ORing
of the two variables, i.e.
A + AB = A + B
Another theorem based on this law is

w
2.4.8

A _A + B i = AB

Idempotent Law
Idempotence means the same value. There are two idempotent laws
A : A : A :g: A = A
A+A+A+g+A = A

2.4.9

Absorption Law
There are two absorption laws
A+A:B = A
A : _A + B i = A

2.4.10

Consensus Theorem
There are two consensus theorems,
AB + AC + BC = AB + AC
_A + B i_A + C i_B + C i = _A + B i_A + C i

Buy Online: shop.nodia.co.in


*Shipping Free*

*Maximum Discount*

GATE STUDY PACKAGE


Electronics & Communication
10 Subject-wise books by R. K. Kanodia
General Aptitude
Engineering Mathematics
Networks
Electronic Devices
Analog Electronics
Digital Electronics
Signals & Systems
Control Systems Communication Systems
Electromagnetics

Page 68
Chap 2

2.4.11

Transposition Theorem
There are two transposition theorems, the first is given as

Boolean Algebra and


Logic Simplification

AB + AC = _A + C i_A + B i
_A + B i : _A + C i = A : C + A : B

2.4.12

De Morgans Theorem
De Morgans theorem gives two of the most powerful laws in Boolean
algebra. These theorems are very useful in simplification of Boolean
expressions,
A+B = A B
AB = A + B

2.4.13

Shannons Expansion Theorem


According to this theorem, any switching expression can be decomposed
with respect to a variable A into two parts, one containing A and the
other containing A . This concept is useful in decomposing complex
system into an interconnection of smaller components.

in
.
o

c
.
ia

f _A, B, C, ....i = A : f _1, B, C...i + A : f _0, B, C, ...i


f _A, B, C, ...i = 8A + f _0, B, C, ...iB : 8A + f _1, B, C, ...iB

2.5

d
o

SIMPLIFICATION OF BOOLEAN EXPRESSIONS USING BOOLEAN


ALGEBRA

n
.
w
w

In Boolean algebra, we have to reduce the Boolean expression into its


simplest form such that the hardware cost reduces efficiently. The basic
rules, laws and theorems of Boolean algebra discussed in this chapter,
are used to simplify Boolean expressions. The following steps are used
to simplify a Boolean expression using Boolean algebra,

METHODOLOGY: TO SIMPLIFY A BOOLEAN EXPRESSION USING


BOOLEAN ALGEBRA

1. Remove all parentheses and multiply all variables.


2. Look for the identical terms. Only one of those terms be retained
and all others skipped. For example,
AB + AB + AB = AB
3. Look for a variable and its complement in the same term. This
term can be removed. For example,
A : BB = A : 0 = 0 ; ABCC = AB : 0 = 0
4. Look for pairs of terms that are identical except for one variable
which may be missing in one of the terms. The larger term can
be removed. For example,
ABC D + ABC = ABC _D + 1i = ABC : 1 = ABC

Buy Online: shop.nodia.co.in


*Shipping Free*

*Maximum Discount*

GATE STUDY PACKAGE

Electronics & Communication

Sample Chapter of

Digital Electronics

(Vol-6, GATE Study Package)

5. Look for pairs of terms which have the same variables, except in
one term a variable is complemented and in other term is it not.
Such terms can be combined into a single terms. For example,

Page 69
Chap 2
Boolean Algebra and
Logic Simplification

ABC D + ABC D = ABC _D + D i = ABC : 1 = ABC


AB _C + D i + AB _C + D i = AB 7_C + D i + _C + D iA = AB : 1 = AB

6. Apply Boolean theorem and laws discussed earlier for further


simplification.

2.5.1

Complement of Boolean Function

i. n

The complement of a Boolean function is obtained in the following


steps:

o
c
.
a

METHODOLOGY: TO OBTAIN COMPLEMENT OF BOOLEAN EXPRESSION

1. Change all the ANDs to ORs and all the ORs to ANDs i.e.,
change all : to + and all + to :

i
d

2. Complement each of the individual variables.

o
n

3. Change all 0s to 1s and 1s to 0s.

2.5.2

.
w
w

Principal of Duality

Duality is a very important property of Boolean algebra. The dual of a


Boolean expression is obtained by replacing all : operations with +
operations, all + operations with : operations, and complementing
all 0s and 1s. The variables are not complemented in this process.
Dual of a function f _A, B, C, ...i is given as

7f _A, B, C,..., 0, 1, + , :iAd = f _A, B, C, ..., 1, 0, : , +i

2.5.3

Relation Between Complement and Dual


For a given Boolean expression f _A, B, C, ...i the relation between its
complement and dual expressions are given as
fc _A, B, C,....i = f _A, B, C, ...i = fd _A, B , C , ...i
fd _A, B, C,...i = f _A, B , C , ...i = fc _A, B , C , ....i
where subscript c represents the complement and subscript d
represents the dual of the given function.
NOTE :

The above relation states that the dual can be obtained by complementing all the
literals in complement function f ^A, B, C, ....h .

2.6

LOGIC GATES

Logic gates are the fundamental building blocks of digital systems.


Logic gates are electronic circuits that perform the most elementary

Buy Online: shop.nodia.co.in


*Shipping Free*

*Maximum Discount*

GATE STUDY PACKAGE


Electronics & Communication
10 Subject-wise books by R. K. Kanodia
General Aptitude
Engineering Mathematics
Networks
Electronic Devices
Analog Electronics
Digital Electronics
Signals & Systems
Control Systems Communication Systems
Electromagnetics

Boolean operations. Before understanding the logic gates, we must


understand the meaning of positive and negative logic.

Page 70
Chap 2
Boolean Algebra and
Logic Simplification

2.6.1

Logic Levels
Inputs and outputs of logic gates can occur only in two levels. These
two levels are termed HIGH and LOW, or TRUE and FALSE, or ON
and OFF, or simply 1 and 0. There are two different ways to assign a
signal value to logic level such as positive logic and negative logic.
1. Positive Logic: If higher of the two voltage levels represents a logic
1 and the lower of the two levels represents a logic 0, then the
logic system is referred to as a positive logic system. Figure 2.1
shows the positive logic system.

Figure 2.1: Positive Logic System

in
.
o

c
.
ia

2.

Negative Logic: If the higher of the two voltage levels represents a


logic 0 and the lower of the two levels represents a logic 1, then
the logic system is referred to as a negative logic system. Figure
2.2 shows the representation of negative logic systems.

d
o

n
.
w
w

Figure 2.2: Negative Logic System

3.

2.6.2

Mixed Logic: In mixed logic, the assignment of logical values to


voltage values is not fixed, and it can be decided by the logic
designers. Mixed logic provides a simplified mechanism for the
analysis and design of digital circuits. The proper use of mixed
logic notation provides logic expressions and logic diagrams that
are analogue to each other. Also, a mixed logic diagram provides
clear information as to the operation of a circuit.

Types of Logic Gates


Logic gates are electronic circuits with a number of inputs and one
output. There are three basic logic gates, namely
1. OR gate,
2.

AND gate,

3.

NOT gate

Buy Online: shop.nodia.co.in


*Shipping Free*

*Maximum Discount*

GATE STUDY PACKAGE

Electronics & Communication

Sample Chapter of

Digital Electronics

(Vol-6, GATE Study Package)

Other logic gates that are derived from these basic gates are
1. NAND gate,

Page 71
Chap 2
Boolean Algebra and
Logic Simplification

2.

NOR gate,

3.

EXCLUSIVE-OR gate,

4.

EXCLUSIVE-NOR gate

AND Gate
An AND gate is a logic circuit with two or more inputs and one output
that performs ANDing operation. The output of an AND gate is HIGH
only when all of its inputs are in the HIGH state. In all other cases, the
output is LOW. For a positive logic systems, it means that the output
of the AND gate is a logic 1 only when all of its inputs are in logic 1
state. In all other cases, the output is logic 0. The logic symbol and
the truth table of a two-input AND gate are shown in Figure 2.3 and
Table 2.6 respectively.

i. n

o
c
.
a

i
d

o
n

Figure 2.3: Logic Symbol of Two-input AND gate

.
w
w

Table 2.6: Truth table of a 2-input AND gate

Input

Output

Y = AB

OR Gate
An OR gate is a logic circuit with two or more inputs and one output
that performs ORing operation. The output of an OR gate is LOW
only when all of its inputs are LOW. For all other possible input
combinations, the output is HIGH. For a positive logic system, the
output of an OR gate is a logic 0 only when all of its inputs are at
logic 0. For all other possible input combinations, the output is a logic
1. The logic symbol and the truth table of a two-input OR gate are
shown in Figure 2.4 and Table 2.7 respectively.

Figure 2.4: Logic Symbol of Two-input OR gate

Buy Online: shop.nodia.co.in


*Shipping Free*

*Maximum Discount*

GATE STUDY PACKAGE


Electronics & Communication
10 Subject-wise books by R. K. Kanodia
General Aptitude
Engineering Mathematics
Networks
Electronic Devices
Analog Electronics
Digital Electronics
Signals & Systems
Control Systems Communication Systems
Electromagnetics

Page 72
Chap 2

Table 2.7: Truth table of a 2-input OR gate

Input

Boolean Algebra and


Logic Simplification

Output

Y = A+B

NOT Gate
A NOT gate, also called an inverter is a one-input, one-output logic
circuit whose output is always the complement of the input. That is,
a LOW input produces a HIGH output, and vice versa. It means that
for a positive logic system, a logic 0 at the input produces a logic 1
at the output, while a logic 1 at the input produces a logic 0 output.
It is also known as a complementing circuit or an inverting circuit. The
logic symbol and the truth table of an inverter are shown in Figure 2.5
and Table 2.8 respectively.

in
.
o

c
.
ia

d
o

Figure 2.5: Symbol for a NOT gate

n
.
w
w

Table 2.8: Truth Table of NOT Gate

Input

Output

Y=A

NAND Gate
The term NAND implies NOT-AND. A NAND gate is equivalent to
AND gate followed by a NOT gate. The standard logic symbol for a
2-input NAND gate is shown in Figure 2.6. This symbol is same as
AND gate symbol except for a small circle (bubble) on its output. This
circle represents the NOT function.

Figure 2.6: Logic symbol of NAND gate

The truth Table 2.9 of a NAND gate is obtained from the truth
Table of an AND gate by complementing the output entries. The
output of a NAND gate is a logic 0 when all its inputs are a logic 1.

Buy Online: shop.nodia.co.in


*Shipping Free*

*Maximum Discount*

GATE STUDY PACKAGE

Electronics & Communication

Sample Chapter of

Digital Electronics

(Vol-6, GATE Study Package)

For all other input combinations, the output is a logic 1. NAND gate
operation is logically expressed as

Page 73
Chap 2
Boolean Algebra and
Logic Simplification

Y = A:B
Table 2.9: Truth Table of a 2-input NAND Gate

Input

Output

Y = AB

i. n

o
c
.
a

NOR Gate

The term NOR implies NOT-OR. A NOR gate is equivalent to OR


gate followed by a NOT gate. The standard logic symbol for a 2-input
NOR gate is shown in Figure 2.7. This symbol is same as OR gate
symbol except for a small circle (bubble) on its output. This circle
represents the NOT function.

i
d

o
n

.
w
w

Figure 2.7: Logic symbol of NOR gate

The truth Table 2.10 of a NOR gate is obtained from the truth
Table of an OR gate by complementing the output entries. The output
of a NOR gate is a logic 1 when all its inputs are logic 0. For all other
input combinations, the output is a logic 0. The output of a two-input
NOR gate is logically expressed as
Y = A+B

Table 2.10: Truth table of a 2-input NOR gate

Input

Output
Y = A+B

Exclusive-OR (XOR) Gate


The Exclusive-OR gate, commonly known as EX-OR gate, is a twoinput, one-output gate. The logic symbol for the Ex-OR gate is shown
in Figure 2.8 and the truth table for a two-input EX-OR operation is
given in Table 2.11.

Buy Online: shop.nodia.co.in


*Shipping Free*

*Maximum Discount*

GATE STUDY PACKAGE


Electronics & Communication
10 Subject-wise books by R. K. Kanodia
General Aptitude
Engineering Mathematics
Networks
Electronic Devices
Analog Electronics
Digital Electronics
Signals & Systems
Control Systems Communication Systems
Electromagnetics

Page 74
Chap 2
Boolean Algebra and
Logic Simplification

Figure 2.8: Symbol for 2-input Ex-OR Gate


Table 2.11: Truth Table of a 2-input Ex-OR Gate

Input

Output

Y = A5B

in
.
o

From the truth table it can be stated that, the output of an EXOR gate is a logic 1 when the two inputs are at different logic and a
logic 0 when the two inputs are at the same logic.

c
.
ia

NOTE :

1.

2.

The exclusive-OR and equivalence gates both can be extended to more than
two inputs. However, multiple-input exclusive OR gates are uncommon from
the hardware standpoint.
For a multiple output-input EX-OR logic function we can conclude that the
output of a multiple-input EX-OR logic function is a logic 1 only when an
odd number of input variables are 1.

d
o

n
.
w
w

Exclusive-NOR (XNOR) Gate

The exclusive-NOR gate, commonly known as Ex-NOR, is an Ex-OR


gate, followed by an inverter. It has two inputs and one output. The
logic symbol for the Ex-NOR gate is shown in Figure 2.9, and the truth
table for the two-input Ex-NOR operation is given in Table 2.12.

Figure 2.9: Symbol for 2-input Ex-NOR Gate


Table 2.12: Truth Table of a 2-input Ex-NOR Gate

Input

Output

Y = A9B

Buy Online: shop.nodia.co.in


*Shipping Free*

*Maximum Discount*

GATE STUDY PACKAGE

Sample Chapter of

Electronics & Communication

Digital Electronics

(Vol-6, GATE Study Package)

The Boolean expression for the Ex-NOR gate is Y = A 5 B . Using


DeMorgans theorem,
A 5 B = AB + AB = AB : AB
= (A + B ) (A + B) = AB + A B

Page 75
Chap 2
Boolean Algebra and
Logic Simplification

The output of a two-input EX-NOR gate is a logic 1 when the


inputs are same and a logic 0 when they are different.
NOTE :

1.

2.

Likewise Ex-OR gates, three or more variable Ex-NOR gates also do not exist.
Normally, multiple-input EX-NOR logic functions can be implemented using
more than one 2-input Ex-NOR gates.
For a multiple output-input EX-NOR logic function we can conclude that the
output of a multiple-input EX-NOR logic function is a logic 1 only when
an even number of input variables are 0. Note if all inputs are 0, then also
output will be 1.

2.7

i. n

UNIVERSAL GATE

o
c
.
a

i
d

NAND and NOR gates are known as universal gates because any of
these two gates is capable of implementing all other gate functions.

2.7.1

o
n

.
w
w

NAND Gate as a Universal Gate

The NAND gate can be used to implement the NOT function, AND
function, the OR function and other functions also as explained below.

The NOT Gate using NAND Gate

An inverter can be made from a NAND gate by connecting all of the


inputs together and creating, in effect, a single common input, as shown
in Figure 2.10, for a two-input NAND gate. Algebraically, we may write
Y = A:B = A:A = A

Figure 2.10: NOT gate using NAND gate

The AND Gate Using NAND Gate


To construct an AND gate from NAND gates, an inverter or a NOT
gate is required to invert the output of a NAND gate. This inversion
cancels out the first inverted operation of NAND gate and the final
result will be AND function as depicted in Figure 2.11. Algebraically,
Y = AB = AB

Figure 2.11: AND Gate using NAND Gate

Buy Online: shop.nodia.co.in


*Shipping Free*

*Maximum Discount*

GATE STUDY PACKAGE


Electronics & Communication
10 Subject-wise books by R. K. Kanodia
General Aptitude
Engineering Mathematics
Networks
Electronic Devices
Analog Electronics
Digital Electronics
Signals & Systems
Control Systems Communication Systems
Electromagnetics

Page 76
Chap 2
Boolean Algebra and
Logic Simplification

The OR Gate using NAND Gate


To construct OR function using only NAND gates, first we transform
the OR function as follows.
Y = A+B = A+B

A=A

(De Morgans Theorem)


= A:B
The above equation is implemented using only NAND gates as
shown in the Figure 2.12.

in
.
o

Figure 2.12: OR Gate using NAND Gate

The NOR Gate Using NAND Gate

c
.
ia

We know that Boolean expression for NOR gate is


(De Morgans Theorem)
Y = A+B = A:B

d
o

= A:B
A=A
The above equation is implemented using only NAND gates, as
shown in the Figure 2.13.

n
.
w
w

Figure 2.13: NOR Gate Using NAND Gate

The Ex-OR Gate using NAND Gate


The Boolean expression for Ex-OR gate is given by
Y = AB + AB
= AB + AB

X=X

Buy Online: shop.nodia.co.in


*Shipping Free*

*Maximum Discount*

GATE STUDY PACKAGE

Sample Chapter of

Electronics & Communication

Digital Electronics

(Vol-6, GATE Study Package)

(De Morgan Theorem)


= _AB i : _AB i
So, five NAND gates are required to implement the Ex-OR gate
as shown in Figure 2.14.

Page 77
Chap 2
Boolean Algebra and
Logic Simplification

i. n

Figure 2.14: Ex-OR Gate using NAND Gate

o
c
.
a

Ex-NOR Gate Using NAND Gate

Ex-NOR gate can be constructed by taking complement of Ex-OR. That


is, we need one more NAND gate to implement the Ex-NOR function.
Figure 2.15 shows Ex-NOR implementation using five NAND gates.

i
d

o
n

.
w
w

Figure 2.15: Ex-NOR Gate Using NAND Gate

2.7.2

NOR Gate as a Universal Gate


Just like the NAND gate, the NOR gate also may be used to implement
all other operations of Boolean algebra. These are explained in following
texts.

NOT Gate Using NOR Gate


In the same way as the NAND gate described above, an inverter can
be made from a NOR gate by connecting all of the inputs together and
creating, in effect, a single common input, as shown in Figure 2.16.
Algebraically,
Y = A+B = A+A = A

Figure 2.16: NOT Gate Using NOR Gate

Buy Online: shop.nodia.co.in


*Shipping Free*

*Maximum Discount*

GATE STUDY PACKAGE


Electronics & Communication
10 Subject-wise books by R. K. Kanodia
General Aptitude
Engineering Mathematics
Networks
Electronic Devices
Analog Electronics
Digital Electronics
Signals & Systems
Control Systems Communication Systems
Electromagnetics

Page 78
Chap 2
Boolean Algebra and
Logic Simplification

OR Gate Using NOR Gate


An OR gate can be created by simply inverting the output of a NOR
gate as shown in Figure 2.17. Algebraically,
Y = A+B = A+B

Figure 2.17: AND Gate Using NOR Gate

AND Gate using NOR Gate


AND function can be generated using three NOR gates. We know that
Boolean expression for AND gate is
Y = A:B

in
.
o

= A:B

A=A

(DeMorgans Theorem)
= A+B
The above equation is implemented using only NOR gates as
shown in the Figure 2.18.

c
.
ia

NAND Gate using NOR Gate

The Boolean expression for NAND gate is

d
o

Y = A:B
= A+B

n
.
w
w

(DeMorgans Theorem)

= A+B
A=A
The above equation is implemented using only NOR gates, as
shown in the Figure 2.19.

Figure 2.18: AND Gate Using NOR Gate

Figure 2.19: NAND gate using NOR gate

Buy Online: shop.nodia.co.in


*Shipping Free*

*Maximum Discount*

GATE STUDY PACKAGE

Electronics & Communication

Sample Chapter of

Digital Electronics

(Vol-6, GATE Study Package)

The Ex-OR Gate using NOR Gate


XOR function may also be implemented by using NOR gates. The ExOR operation is given by,
Y = AB + BA
= AA + AB + BA + BB

Page 79
Chap 2
Boolean Algebra and
Logic Simplification

_AA = BB = 0i

= A _A + B i + B _A + B i = A _A + B i + B _A + B i
= _A + A + B i + _B + A + B i
= _A + A + B i + _B + A + B i
_X = X i
The above expression can be realized using five NOR gates as
shown in Figure 2.20.

i. n

i
d

o
n

.
w
w

o
c
.
a

Figure 2.20: Ex-OR Gate Using NOR Gate

The Ex-NOR Gate using NOR Gate


To implement Ex-NOR gate using NOR gates, we just remove the last
NOR gates from the circuit of Ex-OR gates shown in Figure 2.21.

Figure 2.21: Ex-NOR Gate using NOR Gate

2.8

ALTERNATE LOGIC-GATE REPRESENTATIONS

We have discussed the five basic logic gates (AND, OR, INVERTER,
NAND, and NOR) and the standard symbols used to represent them
in a logic circuit diagram. Most of the logic networks use standard
symbols. But in some networks an alternative set of symbols is used in
addition to the standard symbols. Table 2.13 shows the alternate set of
symbols for the five basic gates.

Buy Online: shop.nodia.co.in


*Shipping Free*

*Maximum Discount*

GATE STUDY PACKAGE


Electronics & Communication
10 Subject-wise books by R. K. Kanodia
General Aptitude
Engineering Mathematics
Networks
Electronic Devices
Analog Electronics
Digital Electronics
Signals & Systems
Control Systems Communication Systems
Electromagnetics

Table 2.13: Alternate Logic Gate Representations

Page 80
Chap 2
Boolean Algebra and
Logic Simplification

Logic

Normal Symbol

Alternate symbol

NOT

AND

OR

NAND

NOR

in
.
o

To convert any normal symbol to its corresponding alternate


symbol, the following steps are used:

c
.
ia

METHODOLOGY: TO CONVERT STANDARD SYMBOL TO ALTERNATE


SYMBOL

Step 1:

d
o

Add bubbles (indication of inversion) at those input or


output points where it is not present.

n
.
w
w

Step 2:

Remove all pre-existing bubbles of the normal symbol, if


there is any at the point (only NOT, NAND and NOR
gates)

Step 3:

If the existing normal logic symbol is AND, change it to


OR, Similarly, if it is OR, then change it to AND. There
is no change for the triangular symbol of NOT gate.

w
2.9

BOOLEAN ANALYSIS OF LOGIC CIRCUITS

A Boolean function may be transformed from an algebraic expression


into a logic diagram using AND, OR and NOT gates. This is also
referred to as AOI logic. Conversely, a logic circuit can be transformed
into Boolean expressions for the analysis.

2.9.1

Converting Boolean Expressions to Logic Diagram


The simplest way to convert a Boolean expression to a logic circuit
is to start with the output and work towards the input. Assume that
the expression Y = AB + AC + ABC is to be realized using AOI logic.
Start with the final expression AB + AC + ABC , we go through
following steps:

Buy Online: shop.nodia.co.in


*Shipping Free*

*Maximum Discount*

GATE STUDY PACKAGE

Electronics & Communication

Sample Chapter of

Digital Electronics

(Vol-6, GATE Study Package)

METHODOLOGY: TO CONVERT BOOLEAN EXPRESSION TO LOGIC


DIAGRAM

Step 1:

The expression Y = AB + AC + ABC contains three


terms (AB , AC , ABC ) which are ORed together. So,
draw an OR gate with three inputs as shown below.

Step 2:

AB must be the output of an AND gate whose inputs are


A and B and AC must be output of an AND gate whose
inputs are A and C . Similarly, ABC must be output of a
3-input AND gate with inputs A , B and C . We introduce
these three AND gates as shown below.

Page 81
Chap 2
Boolean Algebra and
Logic Simplification

i. n

i
d

o
n

.
w
w

Step 3:

o
c
.
a

Now, C must be the output of an inverter whose input is


C and similarly, A will be the output of an inverter whose
input is A. So we put two inverters as shown below.

This is the complete logic diagram of given function

2.9.2

Converting Logic to Boolean Expressions


Any logic circuit, no matter how complex it is, can be described using
Boolean expressions. To derive the Boolean expression for a given
logic circuit, start from the left-most input and work toward the final
output, writing output for each gate. As an example, consider the logic

Buy Online: shop.nodia.co.in


*Shipping Free*

*Maximum Discount*

GATE STUDY PACKAGE


Electronics & Communication
10 Subject-wise books by R. K. Kanodia
General Aptitude
Engineering Mathematics
Networks
Electronic Devices
Analog Electronics
Digital Electronics
Signals & Systems
Control Systems Communication Systems
Electromagnetics

diagram shown in Figure 2.22. We go through the following steps to get


the Boolean expression.

Page 82
Chap 2
Boolean Algebra and
Logic Simplification

Figure 2.22: Logic diagram for which Boolean expression to be determined


METHODOLOGY:
EXPRESSION

2.10

TO

CONVERT

LOGIC

DIAGRAM

TO

BOOLEAN

Step 1:

In the logic diagram shown in Figure 2.22, the output of


left-most OR gate with inputs A and B is _A + B i .

Step 2:

The output of left-most AND gate with inputs C and D


is CD .

Step 3:

The outputs of the OR gate and AND gate are the inputs
of right-most AND gate. Therefore, the expression for
this AND gate is _A + B i : CD , which is the final output
expression for the entire circuit.

in
.
o

c
.
ia

d
o

CONVERTING LOGIC DIAGRAMS TO NAND / NOR LOGIC

n
.
w
w

Since, NAND logic and NOR logic are universal logic system, digital
circuits which are first computed and converted to AOI logic may then
be converted to either NAND logic or NOR logic depending on the
choice.

2.10.1

NAND-NAND Logic
A logic network can be converted into NAND-NAND gate network by
going through following steps:
METHODOLOGY: TO OBTAIN NAND-NAND GATE NETWORK

Step 1:

First draw the circuit in AOI logic i.e., using AND, OR


and NOT gates.

Step 2:

Add a circle (bubble) at the output of each AND gate and


at the inputs to all the OR gates.

Step 3:

Add an inverter on each line that received only one circle


in steps 2, so that the polarity of signals on those lines
remains unchanged from that of the original diagram.

Step 4:

Replace bubbled OR by NAND and each inverter by its


NAND equivalent.

Buy Online: shop.nodia.co.in


*Shipping Free*

*Maximum Discount*

GATE STUDY PACKAGE

Electronics & Communication

Sample Chapter of

2.10.2

Digital Electronics

(Vol-6, GATE Study Package)

NOR-NOR Logic
The procedure of converting an AOI logic to NOR-NOR logic is same
as above except steps 2 and 4.

Page 83
Chap 2
Boolean Algebra and
Logic Simplification

METHODOLOGY: TO OBTAIN NOR-NOR GATE NETWORK

Step 1:

First draw the circuit in AOI logic i.e., using AND, OR


and NOT gates.

Step 2:

Add a circle (bubble) at the output of each OR gate and


at the inputs to all the AND gates.

Step 3:

Add an inverter on each line that received only one circle


in steps 2, so that the polarity of signals on those lines
remains unchanged from that of the original diagram.

Step 4:

Replace bubbled AND by NOR and each inverter by its


NOR equivalent.

i. n

o
c
.
a

o
n

i
d

***********

.
w
w

Buy Online: shop.nodia.co.in


*Shipping Free*

*Maximum Discount*

GATE STUDY PACKAGE


Electronics & Communication
10 Subject-wise books by R. K. Kanodia
General Aptitude
Engineering Mathematics
Networks
Electronic Devices
Analog Electronics
Digital Electronics
Signals & Systems
Control Systems Communication Systems
Electromagnetics

EXERCISE 2.1

Page 84
Chap 2
Boolean Algebra and
Logic Simplification

MCQ 2.1.1

In the following circuit the output Z is

in
.
o

(A) (A + B ) (C + D ) (E + F)
(B) AB + CD + EF
(C) (A + B) (C + D) (E + F)

c
.
ia

(D) AB + CD + EF

MCQ 2.1.2

d
o

In the following circuit, the output X is

n
.
w
w

w
MCQ 2.1.3

(A) MNQ

(B) N (Q + M )

(C) M (Q + N )

(D) Q (M + N )

In the following circuit, the output Z is

(A) AB + (C + D) E

(B) AB (C + D) E

(C) AB + CD + E

(D) AB + CDE

Buy Online: shop.nodia.co.in


*Shipping Free*

*Maximum Discount*

GATE STUDY PACKAGE

Sample Chapter of

MCQ 2.1.4

Electronics & Communication

Digital Electronics

The Boolean expression (X + Y) (X + Y ) (X + Y) is equivalent to


(A) XY
(B) XY
(C) XY

MCQ 2.1.5

(Vol-6, GATE Study Package)

Page 85
Chap 2
Boolean Algebra and
Logic Simplification

(D) XY

In the following circuit, the output Z is

i. n

o
c
.
a

(A) A + B + C

(B) ABC

i
d

(C) AB + BC + AC

MCQ 2.1.6

o
n

Given that AB + AC + BC = AB + AC , then (A + C ) (B + C ) (A + B)


is equivalent to
(A) (A + B ) (A + C )
(B) (A + B) (A + C )

.
w
w

(C) (A + B ) (A + C )

MCQ 2.1.7

MCQ 2.1.8

(D) Above all

(D) (A + B ) (A + C )

In the following circuit the output X is

(A) AB

(B) AB

(C) AB

(D) 0

In the following circuit the output Y is

(A) AB + AB + C

(B) AB + AB + C

(C) AB + AB + C

(D) AB + AB + C

Buy Online: shop.nodia.co.in


*Shipping Free*

*Maximum Discount*

GATE STUDY PACKAGE


Electronics & Communication
10 Subject-wise books by R. K. Kanodia
General Aptitude
Engineering Mathematics
Networks
Electronic Devices
Analog Electronics
Digital Electronics
Signals & Systems
Control Systems Communication Systems
Electromagnetics

Page 86
Chap 2

MCQ 2.1.9

In the following circuit the output Z is

Boolean Algebra and


Logic Simplification

MCQ 2.1.10

MCQ 2.1.11

(A) ABC

(B) ABC

(C) ABC

(D) 0

In the following circuit the output Z is

in
.
o

(A) ABC

(B) AB (C + B)

(C) ABC

(D) AB (C + B)

c
.
ia

d
o

In the following circuit the output Z is

n
.
w
w

(A) ABC

(B) ABC

(C) 0

(D) ABC

w
MCQ 2.1.12

The truth table of a circuit is shown below.


A

The expression for X is


(A) AB + BC + AC + BC

(B) BC + ABC

(C) BC

(D) ABC

Buy Online: shop.nodia.co.in


*Shipping Free*

*Maximum Discount*

GATE STUDY PACKAGE

Electronics & Communication

Sample Chapter of

MCQ 2.1.13

MCQ 2.1.14

Digital Electronics

A + BC is equivalent to
(A) (A + B) (A + C )

(B) A + B

(C) A + C

(D) (A + B ) (A + C )

Boolean Algebra and


Logic Simplification

o
n

(C) (A + B) (B + C )

.
w
w

i. n

o
c
.
a

i
d

(A) (A + B ) (B + C )

(B) (A + B ) (B + C )
(D) Above all

The Boolean expression for the truth table shown is

MCQ 2.1.16

Page 87
Chap 2

The truth table of a circuit is shown below.

The Boolean expression for Z is

MCQ 2.1.15

(Vol-6, GATE Study Package)

(A) B (A + C) (A + C )

(B) B (A + C ) (A + C )

(C) B (A + C ) (A + C )

(D) B (A + C ) (A + C )

The Boolean expression AC + BC is equivalent to


(A) AC + BC + AC
(B) BC + AC + BC + ACB
(C) AC + BC + BC + ABC
(D) ABC + ABC + ABC + ABC

Buy Online: shop.nodia.co.in


*Shipping Free*

*Maximum Discount*

GATE STUDY PACKAGE


Electronics & Communication
10 Subject-wise books by R. K. Kanodia
General Aptitude
Engineering Mathematics
Networks
Electronic Devices
Analog Electronics
Digital Electronics
Signals & Systems
Control Systems Communication Systems
Electromagnetics

Page 88
Chap 2

MCQ 2.1.17

Boolean Algebra and


Logic Simplification

Expression A + AB + A BC + ABC D + A B C DE would be simplified


to
(A) A + AB + CD + E
(B) A + B + CDE
(C) A + BC + CD + DE
(D) A + B + C + D + E

MCQ 2.1.18

The simplified form of a logic function Y = A (B + C (AB + AC )) is


(A) A B
(B) AB
(C) AB

MCQ 2.1.19

(D) AB

The reduced form of the Boolean expression of Y = (AB ) : (AB ) is


(A) A + B

in
.
o

(B) A + B
(C) AB + AB

c
.
ia

(D) A B + AB

MCQ 2.1.20

n
.
w
w

(C) 0

MCQ 2.1.21

MCQ 2.1.22

d
o

If XY + XY = Z then XZ + X Z is equal to
(B) Y
(A) Y

If XY = 0 then X 5 Y is equal to
(A) X + Y

(B) X + Y

(C) XY

(D) XY

If A = 0 in logic expression Z = [A + EF + BC + D] [A + DE + BC + DE ]
, then
(A) Z = 0
(B) Z = 1
(C) Z = BC

MCQ 2.1.23

(D) 1

(D) Z = BC

From a four-input OR gate the number of input condition, that will


produce HIGH output are
(A) 1
(B) 3
(C) 15

(D) 0

Buy Online: shop.nodia.co.in


*Shipping Free*

*Maximum Discount*

GATE STUDY PACKAGE

Sample Chapter of

MCQ 2.1.24

Electronics & Communication

Digital Electronics

(Vol-6, GATE Study Package)

In the following circuit, for which of the following input combination


output will be 1 ?

Page 89
Chap 2
Boolean Algebra and
Logic Simplification

(A) A = 0, B = 0
(B) A = 1, B = 0
(C) A = 0, B = 1

i. n

(D) Either A = 1 or B = 1

MCQ 2.1.25

o
c
.
a

A logic circuit control the passage of a signal according to the following


requirements:
1. Output X will equal A when control input B and C are the same.
2.

i
d

X will remain HIGH when B and C are different.

o
n

The logic circuit would be

.
w
w

w
MCQ 2.1.26

The output of logic circuit is HIGH whenever A and B are both HIGH
as long as C and D are either both LOW or both HIGH. The logic
circuit is

Buy Online: shop.nodia.co.in


*Shipping Free*

*Maximum Discount*

GATE STUDY PACKAGE


Electronics & Communication
10 Subject-wise books by R. K. Kanodia
General Aptitude
Engineering Mathematics
Networks
Electronic Devices
Analog Electronics
Digital Electronics
Signals & Systems
Control Systems Communication Systems
Electromagnetics

Page 90
Chap 2

MCQ 2.1.27

Boolean Algebra and


Logic Simplification

Consider the statements below.


1. If the output waveform from an OR gate is the same as the
waveform at one of its inputs, the other input is being held
permanently LOW.
2.

If the output waveform from an OR gate is always HIGH, one of


its input is being held permanently HIGH.

The statement, which is always true, is


(A) Both 1 and 2
(B) Only 1
(C) 2
(D) None of the above

Common Data For Q. 28 and 29 :


A Boolean function Z = ABC is to be implemented using NAND and
NOR gate. Each gate has unit cost. Only A, B , and C are available.

MCQ 2.1.28

in
.
o

(C) 4 units

MCQ 2.1.29

c
.
ia

If both gates are available then minimum cost is


(A) 2 units
(B) 3 units

d
o

n
.
w
w

If only NAND gates are available, then minimum cost is


(A) 2 units
(B) 3 units
(C) 4 units

MCQ 2.1.30

(D) 6 units

(D) 6 units

In the circuit shown below the LED emits light when

(A) both switches are closed


(B) both switches are open
(C) only one switch is closed
(D) LED does not emit light irrespective of the switch positions

Buy Online: shop.nodia.co.in


*Shipping Free*

*Maximum Discount*

GATE STUDY PACKAGE

Sample Chapter of

MCQ 2.1.31

Electronics & Communication

Digital Electronics

(Vol-6, GATE Study Package)

If the input to the digital circuit shown below consisting of a cascade of


20 XOR gates is X , then the output Y is equal to

Page 91
Chap 2
Boolean Algebra and
Logic Simplification

MCQ 2.1.32

(A) X

(B) X

(C) 0

(D) 1

In the network shown below F can be written as

i. n

o
c
.
a

i
d

(A) X0 X1 X3 X5 + X2 X4 X5 ...Xn - 1 + ...Xn - 1 Xn

o
n

(B) X0 X1 X3 X5 + X2 X3 X4 ...Xn + ...Xn - 1 Xn

.
w
w

(C) X0 X1 X3 X5 ...Xn + X2 X3 X5 ...Xn + ... + Xn - 1 Xn


(D) X0 X1 X3 X5 ...Xn - 1 + X2 X3 X5 ...Xn + ... + Xn - 1 Xn - 2 + Xn

MCQ 2.1.33

The gate G1 and G2 in figure shown below have propagation delays of


10 ns and 20 ns respectively.

If the input Vi makes an abrupt change from logic 0 to 1 at t = t0 then


the output waveform Vo is

[t1 = t0 + 10 ns, t2 = t1 + 10 ns, t3 = t2 + 10 ns]

Buy Online: shop.nodia.co.in


*Shipping Free*

*Maximum Discount*

GATE STUDY PACKAGE


Electronics & Communication
10 Subject-wise books by R. K. Kanodia
General Aptitude
Engineering Mathematics
Networks
Electronic Devices
Analog Electronics
Digital Electronics
Signals & Systems
Control Systems Communication Systems
Electromagnetics

Page 92
Chap 2

MCQ 2.1.34

Which of the following Boolean expressions correctly represents the


relation between P, Q, R and M1

Boolean Algebra and


Logic Simplification

MCQ 2.1.35

(A) M1 = (P OR Q) XOR R

(B) M1 = (P AND Q) XOR R

(C) M1 = (P NOR Q) XOR R

(D) M1 = (P XOR Q) XOR R

If the X and Y logic inputs are available and their complements X


and Y are not available, the minimum number of two-input NAND
required to implement X 5 Y is
(A) 4
(B) 5

c
.
ia

(C) 6

MCQ 2.1.36

in
.
o

(D) 7

d
o

In the negative logic system,


(A) The more negative of the two logic levels represents a logic 1
state

n
.
w
w

(B) The more negative of the two logic levels represents a logic 0
state
(C) All input and output voltage levels are negative
(D) The output is always complement of the intended logic function

w
MCQ 2.1.37
IES EE 1992

Positive logic in a logic circuit is one in which


(A) logic 0 and 1 are represented by 0 and positive voltage respectively
(B) logic 0 and 1 are represented by negative and positive voltages
respectively
(C) logic 0 voltage level is higher than logic 1 voltage level
(D) logic 0 voltage level is lower than logic 1 voltage level

MCQ 2.1.38
IES EC 2002

How is inversion achieved using Ex-OR gate ?


(A) Giving input signal to the two input lines of the gate tied together
(B) Giving input to one input line and logic zero to the other line
(C) Giving input to one input line and logic one to the other line
(D) Inversion cannot be achieved using Ex-OR gate

Buy Online: shop.nodia.co.in


*Shipping Free*

*Maximum Discount*

GATE STUDY PACKAGE

Sample Chapter of

MCQ 2.1.39

Electronics & Communication

Digital Electronics

(Vol-6, GATE Study Package)

Match List-I with List-II and select the correct answer using the codes
given below the lists.
List - I

List - II

a.

A5B = 0

1.

A=
YB

b.

A+B = 0

2.

A=B

c.

A:B = 0

3.

A = 1 or B = 1

d.

A5B = 1

4.

A = 1 or B = 0

MCQ 2.1.40
IES EE 1999

a
3
2
3
2

b
2
3
2
3

c
1
4
4
1

d
4
1
1
4

Boolean Algebra and


Logic Simplification

i. n

Codes :
(A)
(B)
(C)
(D)

Page 93
Chap 2

o
c
.
a

o
n

i
d

Consider the following statements:


(1) A NAND gate is equivalent to an OR gate with its inputs inverted.

.
w
w

(2) A NOR gate is equivalent to an AND gate with its inputs inverted.
(3) A NAND gate is equivalent to an OR gate with its output inverted.
(4) A NOR gate is equivalent to an AND gate with its output inverted.

Which of these statements are correct?


(A) 1 and 2
(B) 2 and 3
(C) 3 and 4

MCQ 2.1.41

(D) 1 and 4

The output (X ) waveform for the combination circuit shown below for
the inputs at A and B (waveform shown in the figure) will be

Buy Online: shop.nodia.co.in


*Shipping Free*

*Maximum Discount*

GATE STUDY PACKAGE


Electronics & Communication
10 Subject-wise books by R. K. Kanodia
General Aptitude
Engineering Mathematics
Networks
Electronic Devices
Analog Electronics
Digital Electronics
Signals & Systems
Control Systems Communication Systems
Electromagnetics

Page 94
Chap 2
Boolean Algebra and
Logic Simplification

in
.
o

MCQ 2.1.42

c
.
ia

d
o

Which of the following represents the correct waveform for X in the


given circuit diagram.

n
.
w
w

Buy Online: shop.nodia.co.in


*Shipping Free*

*Maximum Discount*

GATE STUDY PACKAGE

Sample Chapter of

MCQ 2.1.43

Electronics & Communication

Digital Electronics

(Vol-6, GATE Study Package)

A logic circuit and input waveform to it shown below.

Page 95
Chap 2
Boolean Algebra and
Logic Simplification

i. n

The output waveform is represented by

i
d

o
n

.
w
w

o
c
.
a

MCQ 2.1.44

In a natural food restaurant, fruit is offered for desert but only in


certain combination. One choice is either orange or apple or both.
Another choice is either mango and apple or neither. A third choice
is orange, but if you choose orange, then you must also take banana.
If the fruits are represented by their first alphabet of the name, then
the logical expression that specifies the fruit available for desert in the
simplified form is
(B) M + O
(A) A + B
(C) A + O

MCQ 2.1.45

(D) M + B

The open collector wired circuit shown below functions as

(A) Ex-NOR

(B) AND

(C) Ex-OR

(D) NOR

Buy Online: shop.nodia.co.in


*Shipping Free*

*Maximum Discount*

GATE STUDY PACKAGE


Electronics & Communication
10 Subject-wise books by R. K. Kanodia
General Aptitude
Engineering Mathematics
Networks
Electronic Devices
Analog Electronics
Digital Electronics
Signals & Systems
Control Systems Communication Systems
Electromagnetics

Page 96
Chap 2

MCQ 2.1.46

The simplified form of the expression (w + x) (w l + x + yz l ) (w + y l ) is


(A) w l xy l + wx l + w l yz
(B) w l xy l + wx
(C) xy l + wx + wyz l
(D) xy l + wx + w l yz

MCQ 2.1.47

The elevator door should open if the elevator is stopped, it is level with
the floor, and the timer has not expired, or if the elevator is stopped,
it is level with the floor, and a button is pressed. If
D " Elevator door opens ; S " Elevator is stopped ;
F " Level with floor ; T " Timer expired ; B " Button pressed
Which of the following Boolean expression represents the above
condition ?
(A) D = SFT l + SFB
(B) D = SFT l B
(C) D = SF + T l B
(D) D = (S + F ) T l B

MCQ 2.1.48

The logic circuit shown in the given figure can be minimized to

Boolean Algebra and


Logic Simplification

in
.
o

c
.
ia

d
o

n
.
w
w

w
MCQ 2.1.49

In the following circuit the output Z is

(A) AD (B + C ) + A D

(B) AD (B 5 C ) + A D

(C) AD (B 5 C ) + A D

(D) A D (B 5 C ) + AD

Buy Online: shop.nodia.co.in


*Shipping Free*

*Maximum Discount*

GATE STUDY PACKAGE

Sample Chapter of

MCQ 2.1.50

Electronics & Communication

Digital Electronics

(Vol-6, GATE Study Package)

What does the expression AD + ABCD + ACD + AB + AC D + A B on


minimization result into ?
(A) A + D
(B) AD + A

Page 97
Chap 2
Boolean Algebra and
Logic Simplification

(C) AD
(D) A + D

MCQ 2.1.51

Which one of the following logical operations is performed by the digital


circuit shown below ?

i. n

o
n

(A) NOR

.
w
w

(C) Ex-OR

MCQ 2.1.52
IES EE 1995

i
d

o
c
.
a
(B) NAND
(D) OR

The switching circuit given in the figure an be expressed in binary logic


notation as

(A) L = (A + B) (C + D) E
(B) L = AB + CD + E
(C) L = E + (A + B) (C + D)
(D) L = (AB + CD) E

MCQ 2.1.53
IES EE 2005

Which of the following statements is not correct ?


(A) X + XY = X
(B) X (X + Y) = XY
(C) X + XY = X
(D) ZX + ZXY = ZX + ZY

Buy Online: shop.nodia.co.in


*Shipping Free*

*Maximum Discount*

GATE STUDY PACKAGE


Electronics & Communication
10 Subject-wise books by R. K. Kanodia
General Aptitude
Engineering Mathematics
Networks
Electronic Devices
Analog Electronics
Digital Electronics
Signals & Systems
Control Systems Communication Systems
Electromagnetics

Page 98
Chap 2

MCQ 2.1.54

Consider the given circuit diagram of switching of light from two


different switches.

Boolean Algebra and


Logic Simplification

The input conditions needed to turn on LED is


(B) A = B = 0
(A) A = B = 1
(C) A = 1; B = 0

MCQ 2.1.55
IES EC 1996

(D) Both (A) and (B)

in
.
o

c
.
ia

Which one of the following is the dual-form of the Boolean identity


given below ?
AB + AC = (A + C) (A + B)

d
o

(A) AB + AC = AC + AB

(B) (A + B) + (A + C) = (A + C) (A + B)

n
.
w
w

(C) (A + B) (A + C) = AC + AB
(D) AB + AC = AB + AC + BC

MCQ 2.1.56

What is dual of A + [B + (AC )] + D ?


(A) A + [B (A + C)] + D
(B) A 7B + AC A D
(C) A + [B _A + C i] D

MCQ 2.1.57
IES EE 2004

If x and y are Boolean variables, which one of the following is the


equivalent of x 5 y 5 xy ?
(A) x + y
(B) x + y
(C) 0

MCQ 2.1.58

(D) A [B _A + C i] D

(D) 1

The minimized form of _X + W i_Y 5 Z i + XW l is


(A) (X + W ) (YZ l + Y l Z ) + XW l
(B) (XYZ l + XY l Z + WYZ l + WY l Z )
(C) WYZ l + WY l Z + XW l
(D) WYZ l + WY l Z

Buy Online: shop.nodia.co.in


*Shipping Free*

*Maximum Discount*

GATE STUDY PACKAGE

Sample Chapter of

MCQ 2.1.59

Electronics & Communication

Digital Electronics

(Vol-6, GATE Study Package)

Which of the following logic diagrams represents the original and


simplified expression of the function, F = (x + y) (x + y l ) ?

Page 99
Chap 2
Boolean Algebra and
Logic Simplification

i. n

i
d

o
n

.
w
w

MCQ 2.1.60
IES EC 1992

o
c
.
a

A copy machine generate a stop sign S , to stop the machine operation


and energize and indicates light if according to either of the following
conditions exists:
(1) There is no paper in the paper feeder tray.
(2) The two micro switch in the paper path are activated, indicating
a jam in the paper path.
The presence of paper in the feeder tray is indicated by a high at logic
signal P as shown in figure.

Buy Online: shop.nodia.co.in


*Shipping Free*

*Maximum Discount*

GATE STUDY PACKAGE


Electronics & Communication
10 Subject-wise books by R. K. Kanodia
General Aptitude
Engineering Mathematics
Networks
Electronic Devices
Analog Electronics
Digital Electronics
Signals & Systems
Control Systems Communication Systems
Electromagnetics

Which of the following represents the correct logic circuit so as to get


HIGH output at S ?

Page 100
Chap 2
Boolean Algebra and
Logic Simplification

MCQ 2.1.61

In the following circuit, the motor will turn on when DRIVE = 1

in
.
o

c
.
ia

d
o

n
.
w
w

Which of the following give correct values of A0, A1, A2, A3, A4, A5, A6,
A7, A8 , and A9 in order to move motor ?
(A) A0 = A1 = A2 = A3 = A4 = A5 = A6 = A7 = A8 = A9 = 1

(B) A0 = A1 = A2 = A3 = A4 = A5 = A6 = A9 = 1; A7 = A8 = 0;
(C) A0 = A1 = A2 = A3 = A4 = A5 = A6 = A7 = 1 ; A8 = A9 = 0
(D) A0 = A1 = A2 = A3 = A4 = A5 = A6 = A7 = A8 = 1 ; A9 = 0

MCQ 2.1.62

When two gates with open collector outputs are tied together as shown
in the figure, the output obtained will be

(A) A + B + C + D

(B) A + B + C + D

(C) (A + B ) + (C + D )

(D) (A + B) + (C + D)

Buy Online: shop.nodia.co.in


*Shipping Free*

*Maximum Discount*

GATE STUDY PACKAGE

Sample Chapter of

MCQ 2.1.63

Electronics & Communication

Digital Electronics

(Vol-6, GATE Study Package)

The output of a two level AND-OR gate network is F . What is the


output when all the gates are replaced by NOR gates ?
(A) F
(B) F

Page 101
Chap 2
Boolean Algebra and
Logic Simplification

(C) F D
(D) F

where F D is the dual function of F


Which one of the gates labelled 1,2,3, and 4 in the network shown in
the figure is redundant ?

i. n

(B) 2

.
w
w

(C) 3

(D) 4

i
d

o
n

(A) 1

o
c
.
a

MCQ 2.1.64

GATE EE 2002

For the circuit shown in Figure, the Boolean expression for the output
Y in terms of inputs P , Q , R , and S is

(A) P + Q + R + S
(B) P + Q + R + S
(C) (P + Q ) (R + S )
(D) (P + Q) (R + S)

MCQ 2.1.65

Which of the following circuit implement the Boolean expression


X = AB + CD ?

Buy Online: shop.nodia.co.in


*Shipping Free*

*Maximum Discount*

GATE STUDY PACKAGE


Electronics & Communication
10 Subject-wise books by R. K. Kanodia
General Aptitude
Engineering Mathematics
Networks
Electronic Devices
Analog Electronics
Digital Electronics
Signals & Systems
Control Systems Communication Systems
Electromagnetics

Page 102
Chap 2
Boolean Algebra and
Logic Simplification

in
.
o

c
.
ia

d
o

n
.
w
w

(D) None of the above.

MCQ 2.1.66

Consider the logic circuit shown below.

Using NOR gates only, the circuit can be realised as

Buy Online: shop.nodia.co.in


*Shipping Free*

*Maximum Discount*

GATE STUDY PACKAGE

Sample Chapter of

Electronics & Communication

Digital Electronics

(Vol-6, GATE Study Package)

Page 103
Chap 2
Boolean Algebra and
Logic Simplification

i. n

i
d

o
n

.
w
w

o
c
.
a

w
MCQ 2.1.67

Which of the following represent the correct realization of the given


circuit using NAND gate only ?

Buy Online: shop.nodia.co.in


*Shipping Free*

*Maximum Discount*

GATE STUDY PACKAGE


Electronics & Communication
10 Subject-wise books by R. K. Kanodia
General Aptitude
Engineering Mathematics
Networks
Electronic Devices
Analog Electronics
Digital Electronics
Signals & Systems
Control Systems Communication Systems
Electromagnetics

Page 104
Chap 2
Boolean Algebra and
Logic Simplification

in
.
o

c
.
ia

MCQ 2.1.68

d
o

n
.
w
w

The logic operations of two combinational circuits given in Figure - I


and Figure - II are

(A) Entirely different

(B) Identical

(C) Complementary

(D) Dual

***********

Buy Online: shop.nodia.co.in


*Shipping Free*

*Maximum Discount*

GATE STUDY PACKAGE

Sample Chapter of

Electronics & Communication

Digital Electronics

(Vol-6, GATE Study Package)

EXERCISE 2.2

QUES 2.2.1
IES EC 2003

QUES 2.2.2
IES EC 2006

Page 105
Chap 2
Boolean Algebra and
Logic Simplification

The minimum number of NAND gates required to implement the


Boolean function A + AB + ABC is _____

i. n

The Boolean expression Y (A, B, C ) = A + BC is to be realized using


2-input gates of only one type. The minimum number of gates required
for the realization is _____

o
c
.
a

i
d

QUES 2.2.3

The number of different sets of input conditions that produces a high


output from a five-input OR gate is _____

QUES 2.2.4

A Boolean function of two variables X and Y is defined as follows :


F (0, 0) = F (0, 1) = F (1, 1) = 1; F (1, 0) = 0
Assuming complements of X and Y are not available, a minimum cost
solution for realizing F using 2-input NOR gates and 2-input OR gates
(each having unit cost) would have a total cost of _____ unit.

o
n

.
w
w

w
QUES 2.2.5

To implement Y = ABCD using only two-input NAND gates, minimum


number of requirement of NAND gates is _____

QUES 2.2.6

In circuit shown below, for what input at the terminal A the output
is X = 1 ?

QUES 2.2.7

If X = 1 in logic equation [A + Z {Y + (Z + XY )}][{X + Z (X + Y)} = 1


then Z is _____

Buy Online: shop.nodia.co.in


*Shipping Free*

*Maximum Discount*

GATE STUDY PACKAGE


Electronics & Communication
10 Subject-wise books by R. K. Kanodia
General Aptitude
Engineering Mathematics
Networks
Electronic Devices
Analog Electronics
Digital Electronics
Signals & Systems
Control Systems Communication Systems
Electromagnetics

Page 106
Chap 2

QUES 2.2.8

The minimum number of NOR gates required to implement


A (A + B ) (A + B + C ) is equal to ______

QUES 2.2.9

In the following circuit the output Z is _____

QUES 2.2.10

The number of distinct Boolean expressions of 4 variables is _____

QUES 2.2.11

The number of duals of distinct Boolean expressions of 4 variables is


_____

Boolean Algebra and


Logic Simplification

QUES 2.2.12

in
.
o

c
.
ia

To implement Y = ABCD using two-input NAND gates and NOR


gates, minimum number of requirement of gates is _____

d
o

n
.
w
w

***********

Buy Online: shop.nodia.co.in


*Shipping Free*

*Maximum Discount*

GATE STUDY PACKAGE

Sample Chapter of

Electronics & Communication

Digital Electronics

(Vol-6, GATE Study Package)

EXERCISE 2.3

MCQ 2.3.1
K
SHASHIDHAR
214/12

MCQ 2.3.2
K
SHASHIDHAR
214/13

MCQ 2.3.3
K
SHASHIDHAR
214/14

MCQ 2.3.4

Page 107
Chap 2
Boolean Algebra and
Logic Simplification

The NAND gate can perform the invert function if the inputs are
(A) Connected together
(B) Left open
(C) Either (A) or (B)

(D) None of these

i. n

o
c
.
a

Which of the following gate corresponds to the action of parallel


switches for the input ?
(A) AND
(B) NAND

i
d

(C) OR

o
n

.
w
w

(D) NOR

Which of the following gate corresponds to the action of series switches


for the input ?
(A) AND
(B) NAND
(C) OR

(D) NOR

K
SHASHIDHAR
214/15

MCQ 2.3.5
K
SHASHIDHAR
214/16

Which of the following gate is called universal gate ?


(A) AND
(B) OR
(C) XOR

(D) NAND

In positive logic,
(A) a HIGH = 1, a LOW = 0
(B) a LOW = 1, a HIGH = 0
(C) Only HIGHs are present
(D) Only LOWs are present

MCQ 2.3.6
K
SHASHIDHAR
214/17

The output of an AND gate is LOW


(A) All the time
(B) When any input is LOW
(C) When any input is HIGH
(D) When all inputs are HIGH

Buy Online: shop.nodia.co.in


*Shipping Free*

*Maximum Discount*

GATE STUDY PACKAGE


Electronics & Communication
10 Subject-wise books by R. K. Kanodia
General Aptitude
Engineering Mathematics
Networks
Electronic Devices
Analog Electronics
Digital Electronics
Signals & Systems
Control Systems Communication Systems
Electromagnetics

Page 108
Chap 2
Boolean Algebra and
Logic Simplification

MCQ 2.3.7
K
SHASHIDHAR
214/18

The output of an OR gate is LOW when


(A) All inputs are LOW
(B) Any INPUT is LOW
(C) Any input is HIGH
(D) All inputs are HIGH

MCQ 2.3.8
K
SHASHIDHAR
214/19

MCQ 2.3.9
K
SHASHIDHAR
214/20

MCQ 2.3.10
K
SHASHIDHAR
214/21

If a three-input AND gate has eight input possibilities, how many of


those possibilities will result in a HIGH output?
(A) 1
(B) 2
(C) 7

(D) 8

If a three-input OR gate has eight input possibilities, how many of


those possibilities will result in a HIGH output?
(A) 1
(B) 2
(C) 7

(D) 8

in
.
o

c
.
ia

The output of NOT gate is HIGH when


(A) the input is LOW

d
o

(B) the input is HIGH

n
.
w
w

(C) power is applied to the gates IC


(D) power is removed from the gates IC

MCQ 2.3.11
K
SHASHIDHAR
214/22

The output of an AND gate with three inputs, A, B , and C , is HIGH


when
(A) A = 1, B = 1, C = 0

(B) A = 0 , B = 0 , C = 0
(C) A = 1, B = 1, C = 1
(D) A = 1, B = 0 , C = 1

MCQ 2.3.12
K
SHASHIDHAR
214/23

The output of an OR gate with three inputs, A, B, and C , is LOW


when
(A) A = 0 , B = 0 , C = 0

(B) A = 0 , B = 0 , C = 1
(C) A = 0 , B = 1, C = 1
(D) All of the above

Buy Online: shop.nodia.co.in


*Shipping Free*

*Maximum Discount*

GATE STUDY PACKAGE

Sample Chapter of

MCQ 2.3.13
K
SHASHIDHAR
215/25

MCQ 2.3.14
K
SHASHIDHAR
215/27

Electronics & Communication

Digital Electronics

If a three-input NAND gate has eight input possibilities, how many of


those possibilities will result in a HIGH output?

(A) 1

(B) 2

(C) 7

(D) 8

K
SHASHIDHAR
215/32

Page 109
Chap 2
Boolean Algebra and
Logic Simplification

If a three-input XOR gate has eight input possibilities, how many of


those possibilities will result in a HIGH output?

(A) 2
(B) 4
(C) 6

i. n

(D) 8

MCQ 2.3.15

(Vol-6, GATE Study Package)

o
c
.
a

i
d

A two-input NOR gate is equivalent to a

(A)
(B)
(C)
(D)

negative-OR gate
negative-AND gate
negative-NAND gate
none of the above

o
n

.
w
w

MCQ 2.3.16

K
SHASHIDHAR
215/34

The exclusive-OR gates output is HIGH if

(A) All inputs are low


(B) all inputs are HIGH
(C) the inputs are different
(D) none of the above

MCQ 2.3.17
K
SHASHIDHAR
216/35

The exclusive-NOR gates output is HIGH if


(A) the inputs are the same
(B) one input is High, and the other input is LOW
(C) the inputs are different
(D) none of the above

MCQ 2.3.18
K
SHASHIDHAR
216/38

How many two-input NOR gates does it take to produce a twoinput NAND gate ?
(A) 1
(B) 2
(C) 3

(D) 4

Buy Online: shop.nodia.co.in


*Shipping Free*

*Maximum Discount*

GATE STUDY PACKAGE


Electronics & Communication
10 Subject-wise books by R. K. Kanodia
General Aptitude
Engineering Mathematics
Networks
Electronic Devices
Analog Electronics
Digital Electronics
Signals & Systems
Control Systems Communication Systems
Electromagnetics

Page 110
Chap 2
Boolean Algebra and
Logic Simplification

MCQ 2.3.19
V K PURI
94/1

Boolean algebra can be used to


(A) Simplify any algebraic expressions
(B) Minimize the number of switches in a circuits
(C) Solve the mathematical problems
(D) Perform arithmetic calculations.

MCQ 2.3.20
V K PURI
95/13

De Morgans theorems state that


(A) A + B = A.B and A.B = A.B
.
(B) A + B = A + B and A B = A.B
.
(C) A + B = A.B and A B = A + B
(D) A + B = A + B and A.B = A + B

MCQ 2.3.21
V K PURI
95/16

The gate ideally suited for bit comparison is a


(A) Two input Exclusive NOR gate

c
.
ia

(B) Two input Exclusive OR gate


(C) Two input NAND gate
(D) Two input NOR gate

MCQ 2.3.22
B.R. GUPTA
73/521

in
.
o

d
o

n
.
w
w

A buffer is
(A) always non-inverting
(B) always inverting

(C) inverting or non-inverting


(D) none of above

MCQ 2.3.23

Symbol in figure given below is IEEE symbol for

B.R. GUPTA
83/522

MCQ 2.3.24
B.R. GUPTA
85/522

(A) AND

(B) OR

(C) NAND

(D) NOR

As per Boolean Algebra, inputs can be interchanged in


(A) OR gates
(B) AND gates
(C) both OR and AND gates

(D) none of above

Buy Online: shop.nodia.co.in


*Shipping Free*

*Maximum Discount*

GATE STUDY PACKAGE

Sample Chapter of

MCQ 2.3.25
B.R. GUPTA
94/522

Electronics & Communication

Digital Electronics

In which function is each term known as minterm


(A) SOP
(B) POS
(C) Hybrid

MCQ 2.3.26
MAINI
1/102

MAINI
4/103

(D) Ex-NOR

i. n

In general, logic gates whose all output entries are logic 1 except for
one entry that is logic 0 are
(A) AND, OR

(D) NOR, AND

o
n

.
w
w

o
c
.
a

i
d

(C) NAND, OR

MAINI
8/103

Boolean Algebra and


Logic Simplification

For a certain two-input logic gate, the output is 1 for like inputs and
0 for unlike inputs. The logic gate is
(A) Ex-OR
(B) NAND

(B) NAND, NOR

MCQ 2.3.28

Page 111
Chap 2

(D) both SOP and POS

(C) NOR

MCQ 2.3.27

(Vol-6, GATE Study Package)

A logic gate with four inputs can have


(A) 16 possible input combinations
(B) 4 possible input combinations

(C) 8 possible input combinations


(D) None of these

MCQ 2.3.29
MAINI
4/200

MCQ 2.3.30
MAINI
5/200

The dual of a Boolean expression is A + B . The expression is


(A) A : B
(B) Al : B l
(C) Al + B l
(D) A + B

Complement of complement of Al : B + A : B l is
(A) A : B + Al : B l
(B) _Al + B i : _A + B li
(C) Al : B + A : B l
(D) None of these

MCQ 2.3.31

The operation A : A =
(A) A 2

(B) 2A

(C) 1

(D) A

Buy Online: shop.nodia.co.in


*Shipping Free*

*Maximum Discount*

GATE STUDY PACKAGE


Electronics & Communication
10 Subject-wise books by R. K. Kanodia
General Aptitude
Engineering Mathematics
Networks
Electronic Devices
Analog Electronics
Digital Electronics
Signals & Systems
Control Systems Communication Systems
Electromagnetics

Page 112
Chap 2
Boolean Algebra and
Logic Simplification

MCQ 2.3.32
Chakravorty
M4.10/111

MCQ 2.3.33
Chakravorty
M4.11/111

MCQ 2.3.34
Chakravorty
M4.12/111

MCQ 2.3.35
Chakravorty
M6.1/170

The operation A + A =
(A) A 2

(B) 2A

(C) 0

(D) A

The operation A + A =
(A) 1

(B) A

(C) 0

(D) A

The operation A : A =
(A) 1

(B) A

(C) 0

(A) A

The AND, OR, and NOT gates are called


(A) universal gates

c
.
ia

(B) basic gates


(C) hexadecimal gates
(D) decimal number gates

MCQ 2.3.36

d
o

n
.
w
w

The gate shown in Fig. is an alternative symbol of

Chakravorty
M6.6/171

w
MCQ 2.3.37

in
.
o

(A) AND gate

(B) OR gate

(C) NAND gate

(D) NOR gate

The gate shown in Fig. is an alternative symbol of

Chakravorty
M6.7/171

(A) AND gate


(B) OR gate
(C) NAND gate
(D) NOR gate

Buy Online: shop.nodia.co.in


*Shipping Free*

*Maximum Discount*

GATE STUDY PACKAGE

Sample Chapter of

MCQ 2.3.38

Electronics & Communication

Digital Electronics

(Vol-6, GATE Study Package)

The gate shown in below is an alternative symbol of

Page 113
Chap 2

Chakravorty

Boolean Algebra and


Logic Simplification

M6.8/171

MCQ 2.3.39

(A) AND gate

(B) OR gate

(C) NAND gate

(D) NOR gate

The gate shown below is an alternative symbol of

i. n

Chakravorty
M6.9/171

(A) AND gate

i
d

(B) OR gate
(C) NAND gate

o
n

(D) NOR gate

MCQ 2.3.40
MANDAL
1/72

.
w
w

o
c
.
a

In a positive logic circuit,


(A) Logic 0 and 1 represented 0 V (ground) and positive voltage
_+ VCC i respectively

(B) Logic 0 and 1 represented by negative and positive voltages


respectively
(C) Logic 0 voltage level is higher than logic 1 voltage level
(D) Logic 0 voltage level is lower than logic 1 voltage level

MCQ 2.3.41
MANDAL
2/72

In negative logic, the logic 1 state corresponds to


(A) Ground level
(B) High voltage level
(C) Negative voltage level
(D) Low voltage level

MCQ 2.3.42
MANDAL
3/72

A NAND gate is called a universal logic element because


(A) All digital computers use NAND gates
(B) All the minimisation techniques are applicable for optimum NAND
gate realisation
(C) Every body use this gate
(D) Any logic function can be realised by NAND gates alone

Buy Online: shop.nodia.co.in


*Shipping Free*

*Maximum Discount*

GATE STUDY PACKAGE


Electronics & Communication
10 Subject-wise books by R. K. Kanodia
General Aptitude
Engineering Mathematics
Networks
Electronic Devices
Analog Electronics
Digital Electronics
Signals & Systems
Control Systems Communication Systems
Electromagnetics

Page 114
Chap 2
Boolean Algebra and
Logic Simplification

MCQ 2.3.43
MANDAL
17/73

Boolean algebra is different from ordinary algebra in which way ?


(A) Boolean algebra can represent more than 1 discrete level between
0 and 1.
(B) Boolean algebra have only 2 discrete levels : 0 and 1
(C) Boolean algebra can describe up to levels of logic levels
(D) They are actually the same

MCQ 2.3.44
ANAND KUMAR
10/21

The voltage levels for positive logic system


(A) must necessarily be positive
(B) must necessarily be negative
(C) may be positive or negative
(D) must necessarily be 0 V and 5 V

MCQ 2.3.45
ANAND KUMAR
1/65

in
.
o

Knowledge of binary number system is required for the designers of


computers and other digital systems because
(A) it is easy to learn binary number system

c
.
ia

(B) it is easy to learn Boolean algebra


(C) it is easy to use binary codes

d
o

(D) the devices used in these systems operate in binary

MCQ 2.3.46
ANAND KUMAR
51/136

n
.
w
w

Which of the following statements is true ?


(A) OR and NOT gates are necessary and sufficient for realization of
any logic function.

(B) AND and NOT gates are necessary and sufficient for realization of
any logic function.
(C) NAND gates are not sufficient to realize any logic function.
(D) NOR gates are sufficient to realize any logic function.

MCQ 2.3.47

For the gate shown in the figure, the output will be HIGH

(A) if and only if both inputs are HIGH


(B) if and only if both the inputs are LOW
(C) if one of the inputs is LOW
(D) if one of the inputs is HIGH

Buy Online: shop.nodia.co.in


*Shipping Free*

*Maximum Discount*

GATE STUDY PACKAGE

Sample Chapter of

MCQ 2.3.48
ANAND KUMAR
25/67

Electronics & Communication

Digital Electronics

(Vol-6, GATE Study Package)

In a digital computer binary subtraction is performed


(A) in the same way as we perform subtraction in decimal number
system
(B) using 2s complement method

Page 115
Chap 2
Boolean Algebra and
Logic Simplification

(C) using 9s complement method


(D) using 10s complement method

MCQ 2.3.49
ANAND KUMAR
63/137

ANAND KUMAR
65/138

(D) AND, OR, and NOT

o
c
.
a

Which of the following operations is commutative but not associative ?


(A) AND
(B) NOR
(C) OR

MCQ 2.3.51

i
d

o
n

.
w
w

(D) X-OR

A + AB + ABC + ABCD + ABCDE + ... =


(A) 1
(B) A
(C) A + AB

(D) AB

MCQ 2.3.52
ANAND KUMAR
2/192

MCQ 2.3.53

A + AB + A BC + A B C D + .... =
(A) A + B + C + ...

(B) A + B + C + D + ...

(C) 1

(D) 0

The number of table entries needed for a five input logic circuit is
(A) 4
(B) 8
(C) 16

MCQ 2.3.54
ANAND KUMAR
15/192

i. n

(C) NOR

MCQ 2.3.50

ANAND KUMAR
1/192

The most suitable gate to check whether the number of 1s in a digital


word is even or odd is
(A) X-OR
(B) NAND

(D) 32

The dual of a Boolean expression is obtained by


(A) interchanging all 0s and 1s
(B) interchanging all 0s and 1s, all + and :signs
(C) interchanging all 0s and 1s, all + and : signs and complementing
all the variables
(D) interchanging all + and : signs and complementing all the
variables

Buy Online: shop.nodia.co.in


*Shipping Free*

*Maximum Discount*

GATE STUDY PACKAGE


Electronics & Communication
10 Subject-wise books by R. K. Kanodia
General Aptitude
Engineering Mathematics
Networks
Electronic Devices
Analog Electronics
Digital Electronics
Signals & Systems
Control Systems Communication Systems
Electromagnetics

Page 116
Chap 2
Boolean Algebra and
Logic Simplification

MCQ 2.3.55
ANAND KUMAR
16/192

The complement of a Boolean expression is obtained by


(A) interchanging all 0s and 1s
(B) interchanging all 0s and 1s, all + and : signs
(C) interchanging all 0s and 1s, all + and : signs and complementing
all the variables
(D) interchanging all + and : signs and complementing all the
variables

MCQ 2.3.56
SEDHA
7/118

All Boolean expressions can be implemented with


(A) NAND gates only
(B) NOR gates only
(C) combinations of NAND and NOR gates
(D) Any of these

MCQ 2.3.57

in
.
o

Which of the following logic gates will have an output of 1 ?

c
.
ia

d
o

MCQ 2.3.58
SEDHA
1/183

MCQ 2.3.59
SEDHA
15/184

MCQ 2.3.60
KHARATE
7/197

n
.
w
w

Boolean algebra is essentially based on


(A) symbols
(B) logic
(C) truth

(D) numbers

X + XY is reduced to
(A) X

(B) X + Y

(C) X + Y

(D) X + Y

A carry look ahead adder is frequently used for addition, because it


(A) is faster
(B) is more accurate
(C) uses fewer gates

(D) costs less

***********

Buy Online: shop.nodia.co.in


*Shipping Free*

*Maximum Discount*

GATE STUDY PACKAGE

Sample Chapter of

Electronics & Communication

Digital Electronics

(Vol-6, GATE Study Package)

SOLUTIONS 2.1

SOL 2.1.1

Page 117
Chap 2
Boolean Algebra and
Logic Simplification

Correct option is (B).


From the Boolean properties, we know that

i. n

o
c
.
a

So, its equivalent logic will be

i
d

o
n

i.e.
AND-Invert = Invert-OR
By converting AND Invert logic to equivalent Invert-OR logic in the
given circuit diagram, we get

.
w
w

So, the output Z is


Z = AB + CD + EF
ALTERNATIVE METHOD :

Expression of output can be directly obtained from given circuit as


Z = _AB i_CD i_EF i
Using De-Morgans theorem, we have
Z = AB + CD + EF

SOL 2.1.2

Correct option is (D).


From the Boolean properties, we know that

Buy Online: shop.nodia.co.in


*Shipping Free*

*Maximum Discount*

GATE STUDY PACKAGE


Electronics & Communication
10 Subject-wise books by R. K. Kanodia
General Aptitude
Engineering Mathematics
Networks
Electronic Devices
Analog Electronics
Digital Electronics
Signals & Systems
Control Systems Communication Systems
Electromagnetics

Page 118
Chap 2
Boolean Algebra and
Logic Simplification

So, its equivalent logic will be

i.e.
AND-Invert = Invert-OR
Applying the property, we have the modified logic circuit as

in
.
o

c
.
ia

d
o

So, the output X is

X = MNQ + MNQ + M NQ

n
.
w
w

w
SOL 2.1.3

= MQ _N + N i + M NQ
= MQ + M NQ
= Q _M + M N i
= Q _M + N i

Correct option is (A).


We convert the AND-Invert logic to equivalent Invert-OR logic as

or

Buy Online: shop.nodia.co.in


*Shipping Free*

*Maximum Discount*

GATE STUDY PACKAGE

Sample Chapter of

Electronics & Communication

Digital Electronics

(Vol-6, GATE Study Package)

Page 119
Chap 2
Boolean Algebra and
Logic Simplification

So, the output Z is given as


Z = AB + E _C + D i

SOL 2.1.4

i. n

o
c
.
a

Correct option is (C).


By using the Boolean properties, we minimize the given Boolean
expression as

_X + Y i_X + Y i_X + Y i = _X + Y i_X : X + XY + X Y + Y : Y i

o
n

i
d

SOL 2.1.5

.
w
w

= _X + Y i_XY + X Y i
= XY + XY
= XY

Correct option is (D).


From the given logic diagram, expression of the output can be written
as

Z = A + _AB + BC i + C
= A+A+B+B+C+C
= A+B+C
4
= ABC
From the above logic function, we can observe that options (A) and (B)
are matched. Now, we check the expression given in option (C).

Z = AB + BC + AC
= A+B+B+C+A+C
= A+B+C
Hence, all the options are same, and equal to the output Z of the given
logic circuit.

SOL 2.1.6

Correct option is (B).


Given that
AB + AC + BC = AB + AC
From Consensus Theorem, when a particular variable is associated
with some variable and its complement is associated with another
variable and next term is formed by the leftover variables, then the last
term becomes redundant.

Buy Online: shop.nodia.co.in


*Shipping Free*

*Maximum Discount*

GATE STUDY PACKAGE


Electronics & Communication
10 Subject-wise books by R. K. Kanodia
General Aptitude
Engineering Mathematics
Networks
Electronic Devices
Analog Electronics
Digital Electronics
Signals & Systems
Control Systems Communication Systems
Electromagnetics

AB + AC + BC
S

Page 120
Chap 2

= AB + AC

redundant
term

Boolean Algebra and


Logic Simplification

Its dual also exists. Taking dual of the expression, we get

_A + B i_A + C i_B + C i = _A + B i_A + C i

Hence,

SOL 2.1.7

_A + C i_B + C i_A + B i = _A + B i_A + C i

Correct option is (A).


Expression of output for the circuit is given by
X = _A 5 B i : _A + B i
= _AB + AB i_AB i
= AB

SOL 2.1.8

Y = _A 5 B i : C

c
.
ia

= _AB + AB i : C
= _AB + AB i + C
= _AB + A B i + C
= A B + AB + C

SOL 2.1.9

in
.
o

Correct option is (B).


Expression of the output for the circuit is given by

d
o

{Using De Morgans theorem}


{A 9 B = A 5 B or A 5 B = A 9 B }

n
.
w
w

Correct option is (C).


Expression of the output Z for the circuit is given by
Z = A : _A + A i : B : C
= ABC

{A + A = 1}

Correct option is (A).


Expression of the output Z for the circuit is given by
Z = _A : B i : _B + C i
= A : B : B + ABC
= ABC

{B : B = 0 }

w
SOL 2.1.10

SOL 2.1.11

Correct option is (A).


The expression of the output Z for the circuit is given by
Z = _A + B i : BC
{Using De-Morgans theorem}
= AB : BC
= ABC

Buy Online: shop.nodia.co.in


*Shipping Free*

*Maximum Discount*

GATE STUDY PACKAGE

Electronics & Communication

Sample Chapter of

SOL 2.1.12

Digital Electronics

(Vol-6, GATE Study Package)

Correct option is (C).


Expression for X from given table is obtained by writing logic for X
corresponding to 0 output. i.e.
X = A BC + ABC
= _A + Ai BC
= BC

SOL 2.1.13

Page 121
Chap 2
Boolean Algebra and
Logic Simplification

{A + A = 1}

Correct option is (A).


Given expression is A + BC . Using distributive law, we have
A + BC = _A + B i_A + C i
This law states that ANDing of several variables and ORing the result
with a single variable is equivalent to ORing that single variable with
each of the several variables and then ANDing the sums. It can be
verified from the table below.

i. n

SOL 2.1.14

A+C

_A + B i_A + C i

A + BC

A+B

o
n
0

.
w
w

i
d

o
c
.
a

Correct option is (B).


The expression of Z from given truth table can be written for logic
1 or indirectly we can solve for Z and then take complement. The
expression for Z is given by , writing logic expression for 0 output as,
Z = A BC
Taking complement, we get
{Using De-Morgans theorem}
Z = Z = A BC
or
Z = A+B+C
Now, we check the result for the given options. From expression given
in option (B), we get same minimized result.
_A + B i_B + C i = _A + B i + _B + C i

= A+B+C

Buy Online: shop.nodia.co.in


*Shipping Free*

*Maximum Discount*

GATE STUDY PACKAGE


Electronics & Communication
10 Subject-wise books by R. K. Kanodia
General Aptitude
Engineering Mathematics
Networks
Electronic Devices
Analog Electronics
Digital Electronics
Signals & Systems
Control Systems Communication Systems
Electromagnetics

Page 122
Chap 2

SOL 2.1.15

Boolean Algebra and


Logic Simplification

Correct option is (A).


From truth table, the expression of the function f is given by
f = ABC + ABC
= B _AC + AC i
= B _A + C i_A + C i

SOL 2.1.16

Correct option is (D).


The given Boolean expression is rewritten as
or

AC + BC = AC _B + B i + BC _A + A i
AC + BC = ABC + ABC + ABC + ABC

SOL 2.1.17

Correct option is (D).


The given expression is
A + AB + A BC + A B C D + A B C DE
On simplification of the expression using Boolean algebra, we get
A + AB + A BC + A B C D + A B C DE

in
.
o

c
.
a

= A + A 8B + B #C + C _D + DE i-B
Using redundant literal rule, we have

...(1)

i
d

A + AB = A + B
or
A + A _B + C i = A + B + C
Applying this rule in equation (1), we get

o
n
.
w
w

A + AB + A BC + A B C D + A B C DE

w
SOL 2.1.18

= A + A 7B + B #C + C _D + E i-A
= A + A #B + B _C + D + E i= A + A _B + C + D + E i
= A+B+C+D+E

Correct option is (B).


Given logic function is

Y = A #B + C _AB + AC iOn simplification, we get

Y = AB + AC 7_A + B i_A + C iA
= AB + AC _A + A C + A B + B C i
= AB

SOL 2.1.19

Correct option is (D).


Given logic expression is
Y = _AB i : _AB i
On simplification by using Boolean algebra, we get

Buy Online: shop.nodia.co.in


*Shipping Free*

*Maximum Discount*

GATE STUDY PACKAGE

Electronics & Communication

Sample Chapter of

Digital Electronics

(Vol-6, GATE Study Package)

Y = _AB i : _AB i (Using De-Morgans theorem)


= _A + B i_A + B i
= _AA + A B + AB + BB i
= A B + AB

SOL 2.1.20

Page 123
Chap 2
Boolean Algebra and
Logic Simplification

Correct option is (B).


Given that
Z = XY + XY
So, we simplify the given function as
XZ + X Z = X _XY + XY i + X _XY + XY i
= X _XY + X Y i + XY
= XY + XY

o
c
.
a

= Y _X + X i
=Y

SOL 2.1.21

Correct option is (A).


Given that
XY = 0
Since, we know that

i
d

o
n

.
w
w

i. n

X 5 Y = XY + XY

X5Y = X9Y
So, by using the given condition, we get

w
or

SOL 2.1.22

X 5 Y = XY + XY = _XY + X Y i
X5Y = X Y = X+Y

(XY = 0 )

Correct option is (C).


Rearranging the given expression, we get

Z = 7A + EF + BC + DA7A + D E + BC + D F A
= 7A + BC + EF + DA7A + BC + D _E + F iA

= 7_A + BC i + _EF + D iA7_A + BC i + _D + EF iA


Let
A + BC = X
and
EF + D = Y
So, we may write
Z = _X + Y i_X + Y i
= X + XY + XY
= X 71 + Y + Y A
=X
= A + BC

Given that A = 0 , then


Z = BC

Buy Online: shop.nodia.co.in


*Shipping Free*

*Maximum Discount*

GATE STUDY PACKAGE


Electronics & Communication
10 Subject-wise books by R. K. Kanodia
General Aptitude
Engineering Mathematics
Networks
Electronic Devices
Analog Electronics
Digital Electronics
Signals & Systems
Control Systems Communication Systems
Electromagnetics

Page 124
Chap 2

SOL 2.1.23

Boolean Algebra and


Logic Simplification

Correct option is (C).


There are Four inputs (let A, B , C , D ) to a OR gate. So, we have
Total no. of input conditions = 2 4 = 16
Now, the output of OR gate is given by
Z = A+B+C+D
i.e. the output is High, if any one out of 4 input is high; and output
will Low, only for one condition (0 0 0 0). Thus, output will high for
15 input conditions.

SOL 2.1.24

Correct option is (C).


The expression for the output of given circuit is
F = 7_A + B i + A + _A + B iA : 7_A + B i + A A
Let
A + B = X and A + _A + B i = Y
So, the expression for output can be minimized as

in
.
o

F = _Y + X i Y
= Y + XY = Y _1 + X i = Y

= A + _A + B i
= A : _A + B i
= AA + AB
= AB
Thus, we may conclude that
F = 1 for A = 0 and B = 1

c
.
ia

d
o

SOL 2.1.25

n
.
w
w

Correct option is (A).


We check the given requirements for the circuits given in the options.
Consider the circuit of option (A).

1. For B = C :
P = B5C = 0
and
X = A+0 = A
i.e. Output X will equal A when control input B and C are the same.
2. For B ! C :
P = B5C = 1
and
X = A+1 = 1
i.e. X will remain HIGH when B and C are different.
Hence, the circuit satisfies both the requirements.

Buy Online: shop.nodia.co.in


*Shipping Free*

*Maximum Discount*

GATE STUDY PACKAGE

Sample Chapter of

SOL 2.1.26

Electronics & Communication

Digital Electronics

(Vol-6, GATE Study Package)

Correct option is (A).


We check the circuits in given option for the required condition. The
output of the logic circuit will be high only when both inputs of last
AND are high. Now, we have the two conditions:
1. Given that A and B are both High. For A and B as logic High,
one input of last AND gate is high for the circuits given in options
(A) and (B).
2.

Page 125
Chap 2
Boolean Algebra and
Logic Simplification

Given that C and D are either both LOW or both HIGH. For the
circuit given in option (A), if C and D inputs are either both high
or both low, i.e. C = D applied to XNOR gate then
C 9 D = 1 for C = D
i.e. another input of last AND gate will be High.

i. n

Thus, the circuit given in option (A) is HIGH whenever A and B are
both HIGH as long as C and D are either both LOW or both HIGH.

SOL 2.1.27

o
c
.
a

i
d

Correct option is (D).


Consider a 2-input OR gate shown below.

o
n

.
w
w

Now, we check the correctness of the given two statements.


1. Given that output waveform _X i is same as the any one input
(let A). For this condition, we may have the following two state
diagrams.

From the above state diagram, we may observe that it is not


necessary that B should be permanently Low to satisfy the
required condition. Therefore, statement-1 is False.
2.

Given that output wave form _X i is always high. Again, we may


draw the state diagram for the condition.

Buy Online: shop.nodia.co.in


*Shipping Free*

*Maximum Discount*

GATE STUDY PACKAGE


Electronics & Communication
10 Subject-wise books by R. K. Kanodia
General Aptitude
Engineering Mathematics
Networks
Electronic Devices
Analog Electronics
Digital Electronics
Signals & Systems
Control Systems Communication Systems
Electromagnetics

From the above state diagram, we observe that it is not necessary


that any one input should be permanently high. Therefore,
statement-2 is False.

Page 126
Chap 2
Boolean Algebra and
Logic Simplification

SOL 2.1.28

Correct option is (A).


Given Boolean function is
Z = ABC
To obtain the Boolean expression in form of NAND gate output and
NOR gate output, we rewrite the expression as
Z = ABC = ACB = AC + B
Let
AC = D
then
Z = D+B
Therefore, one NAND gate and one NOR gate is required to implement
the Boolean function as shown below.

in
.
o

c
.
ia

Hence, the minimum cost for the implementation is 2 units.

SOL 2.1.29

d
o

n
.
w
w

Correct option is (C).


Given Boolean function,

Z = ABC
To implement the function using using only NAND gates, we draw the
logic circuit as

Now, we convert each gate to its NAND implementation.

Thus, the minimum cost for implementation of the function using


NAND gate will be 5 units.

Buy Online: shop.nodia.co.in


*Shipping Free*

*Maximum Discount*

GATE STUDY PACKAGE

Sample Chapter of

SOL 2.1.30

Electronics & Communication

Digital Electronics

(Vol-6, GATE Study Package)

Correct option is (D).


For the LED circuit, we know the following points:
1. Output of NAND gate must be Low for LED to emit light.
2.

Both inputs to NAND must be High for Low output.

3.

If any one of the switches is closed, output of AND gate will be


Low.

4.

If Both switches are open, output of XOR gate will be Low.

Page 127
Chap 2
Boolean Algebra and
Logic Simplification

So there can not be both input High to NAND. Therefore, LED does
not emit light irrespective of the switch positions.

SOL 2.1.31

i. n

Correct option is (D).


For the given circuit, we have
Output of 1 st XOR = X : 1 + X : 1 = X
Output of 2 nd XOR = X 5 X
= X X + XX = 1
rd
Output of 3 XOR = 1 5 X
and so on.
= X:1+X:1 = X
Hence, after 2, 4, 6, 8, .......20 XOR (i.e. even number of XOR gates),
output will be 1.

i
d

o
n

SOL 2.1.32

.
w
w

o
c
.
a

Correct option is (C).


For the given network, we obtain
Output of gate 1 = X0 X1

Output of gate 2 = X0 X1 + X2
Output of gate 3 = _X0 X1 + X2i X3
= X0 X1 X3 + X2 X3
Similarly, we may deduce
Output of gate 4 = X0 X1 X3 + X2 X3 + X4
Output of gate 5 = _X0 X1 X3 + X2 X3 + X4i X5
= X0 X1 X3 X5 + X2 X3 X5 + X4 X5
Hence, the output of gate n would be
F = X0 X1 X3 X5 .........Xn + X2 X3 X5 .........Xn + X4 X5 X7 ..........Xn + ........ + Xn - 1 Xn

SOL 2.1.33

Correct option is (C).


Given that G1 has delay of 10 ns and G2 has delay of 20 ns. Let output
of G1 is X . So, we get the output waveform for the given circuit as
shown below.

Buy Online: shop.nodia.co.in


*Shipping Free*

*Maximum Discount*

GATE STUDY PACKAGE


Electronics & Communication
10 Subject-wise books by R. K. Kanodia
General Aptitude
Engineering Mathematics
Networks
Electronic Devices
Analog Electronics
Digital Electronics
Signals & Systems
Control Systems Communication Systems
Electromagnetics

Page 128
Chap 2
Boolean Algebra and
Logic Simplification

SOL 2.1.34

Correct option is (D).


From the circuit diagram, we have
X = PQ
Y = P+Q

and

and
Hence,

SOL 2.1.35

in
.
o

c
.
ia

Z = X : Y = _PQ i_P + Q i
= _P + Q i_P + Q i
= PQ + PQ = P 5 Q
M1 = Z 5 R

d
o

M1 = _P 5 Q i 5 R
M1 = _P XOR Q i XOR R

n
.
w
w

Correct option is (A).


XOR logic using 2-input NAND gates is implemented as

Now, we may prove that the above logic circuit implements an XOR
gate
Z = $_XY i X . : $_XY i Y .
= _XY i X + _XY i Y
= _X + Y i X + _X + Y i Y
= XY + XY = X 5 Y
Thus, 4 NAND gates are required

Buy Online: shop.nodia.co.in


*Shipping Free*

*Maximum Discount*

GATE STUDY PACKAGE

Sample Chapter of

SOL 2.1.36

SOL 2.1.37

Electronics & Communication

Digital Electronics

Correct option is (A).


In negative logic system, Low-level or more negative represent the logic
1 and HIGH level or less negative represent the logic 0, as illustrated
in figure below.

Page 129
Chap 2
Boolean Algebra and
Logic Simplification

i. n

Correct option is (D).


In positive logic system Low-level or more negative represents the logic
0 and HIGH level or less negative represents the logic 1.

.
w
w

o
c
.
a

i
d

o
n

SOL 2.1.38

(Vol-6, GATE Study Package)

Correct option is (C).


Ex-OR gate output is given by

(A and B are inputs)


Y = A5B
= AB + AB
To make inversion of a input using XOR gate, we consider one input,
let A. So, we must have the output
Y =A
For the required output, we should take another input at logic 1 (High). i.e.
Y = A:1+A:1 = A

SOL 2.1.39

Correct option is (B).


We know that, XOR output is logic 1 when both inputs are not equal
and logic 0 when both inputs are same. Hence, we have
A 5 B = 0 for A = B
i.e. (a " 2 ) and _d " 1i
A 5 B = 1 for A ! B
Again, XNOR output become logic 0, if any one input of XNOR is logic
1 (high), i.e.
i.e. _b " 3i
A + B = 0 for A = 1 or B = 1 or A = B = 1
Also, we have
i.e. _c " 4i
A : B = 0 for A = 1 or B = 0
Therefore, the correct match in the list is
(a " 2 ), _b " 3i , _c " 4i , _d " 1i

Buy Online: shop.nodia.co.in


*Shipping Free*

*Maximum Discount*

GATE STUDY PACKAGE


Electronics & Communication
10 Subject-wise books by R. K. Kanodia
General Aptitude
Engineering Mathematics
Networks
Electronic Devices
Analog Electronics
Digital Electronics
Signals & Systems
Control Systems Communication Systems
Electromagnetics

Page 130
Chap 2

SOL 2.1.40

Boolean Algebra and


Logic Simplification

Correct option is (A).


A NAND gate output can be expressed as
Y = A : B = A + B = A OR B
So, NAND gate is equivalent to an OR gate with its inverted inputs.

Again, the NOR gate output can be expressed as


Y = A + B = A : B = A _ANDi B
Hence, NOR gate is equivalent to an AND gate with its inverted input.

in
.
o

Therefore, statements (1) and (2) are correct.

SOL 2.1.41

c
.
ia

Correct option is (B).


For given logic circuit, expression for output X is

X = _A + B i : B = _A + B i + B
= ^A + B h + B = A + B
Output waveform for the given input waveforms is

d
o

n
.
w
w

Buy Online: shop.nodia.co.in


*Shipping Free*

*Maximum Discount*

GATE STUDY PACKAGE

Electronics & Communication

Sample Chapter of

SOL 2.1.42

Digital Electronics

(Vol-6, GATE Study Package)

Correct option is (C).


The expression of the output for the given logic circuit is

Page 131
Chap 2
Boolean Algebra and
Logic Simplification

X = AB + A B = A 9 B
So, we may conclude that
X = A 9 B = 1 for A = B
= 0 for A ! B
Therefore, we obtain the output waveform for the given input waveforms
as

i. n

i
d

o
n

SOL 2.1.43

.
w
w

o
c
.
a

Correct option is (D).


The expression of the output for the given logic circuit is
X = _A + B i + B
= _A + B i : B

= _A : B i : B
=0
Hence, the output for the circuit will remain zero irrespective of the
input.

SOL 2.1.44

Correct option is (C).


According to the given problem, we represent the fruits as
A = apple; B = banana; M = Mango; O = orange
So, the logical expression that specifies the fruit available for desert is
f _A, B, M, O i = _1st Choicei + _2nd Choicei + _3rd Choicei
= _O + A + OAi + _MAi + _OB i
= _A + O i + MA + BO
= A _1 + M i + O _1 + B i
= A+O

Buy Online: shop.nodia.co.in


*Shipping Free*

*Maximum Discount*

GATE STUDY PACKAGE


Electronics & Communication
10 Subject-wise books by R. K. Kanodia
General Aptitude
Engineering Mathematics
Networks
Electronic Devices
Analog Electronics
Digital Electronics
Signals & Systems
Control Systems Communication Systems
Electromagnetics

Page 132
Chap 2

SOL 2.1.45

Correct option is (C).


The wired AND in open-collector is given by

Boolean Algebra and


Logic Simplification

Since, for the given circuit, we have


C = A and D = B
Hence, the output of the circuit is given as
F = AB : A B = AB + AB
= A 5 B = A _XOR i B

SOL 2.1.46

in
.
o

Correct option is (C).


Given Boolean expression is
_w + x i_w l + x + yz li_w + y li
On simplification, we obtain
_w + x i_w l + x + yz li_w + y li = _w l + x + yz li_w + xy li
= ww l + w l xy l + xw + xxy l + yz l w + yz l xy l
= w l xy l + xy l + xw + yz l w

c
.
ia

d
o

SOL 2.1.47

n
.
w
w

= _w l + 1i xy l + w _x + yz li
= xy l + wx + wyz l

Correct option is (A).


The elevator door will open in the following two cases.
Case 1: If the elevator is stopped, it is level with the floor, and the
timer has not expired.
Since, we have the representations
Elevator is stopped = S ;
Level with the floor = F ;
Time has not expired = T l
So, the given condition is expressed as
X1 = SFT l
Case 2: If the elevator is stopped, it is level with the floor, and a button
is pressed.
Again, we have the representations
Elevator is stopped = S
and
Level with the floor = F
and
Button is pressed = B
So, the given condition is expressed as

Buy Online: shop.nodia.co.in


*Shipping Free*

*Maximum Discount*

GATE STUDY PACKAGE

Sample Chapter of

Electronics & Communication

Digital Electronics

(Vol-6, GATE Study Package)

X2 = SFB
Since, the door will be open is case-1 or case-2. Therefore, we may
express the condition for elevator door to be open as
D = X1 OR X2
D = SFT l + SFB

or

SOL 2.1.48

Page 133
Chap 2
Boolean Algebra and
Logic Simplification

Correct option is (D).


Given logic circuit is

i. n

o
c
.
a

i
d

So, the output Z is given by

Z = X + X + Y = X : X + Y = X : _X + Y i
= X + XY = X _1 + Y i = X = X
In option (D), the circuit provides the output X as shown below.

o
n

.
w
w

Hence, the circuit given in option (D) is minimized form of the logic
circuit.

w
SOL 2.1.49

Correct option is (B).


We redraw the given logic circuit as

Output Z of the logic circuit is


Z = AB C D + ABCD + A D

Buy Online: shop.nodia.co.in


*Shipping Free*

*Maximum Discount*

GATE STUDY PACKAGE


Electronics & Communication
10 Subject-wise books by R. K. Kanodia
General Aptitude
Engineering Mathematics
Networks
Electronic Devices
Analog Electronics
Digital Electronics
Signals & Systems
Control Systems Communication Systems
Electromagnetics
= AD ^B C + BC h + A D

Page 134
Chap 2

= AD _B 9 C i + A D

Boolean Algebra and


Logic Simplification

= AD _B 5 C i + A D

SOL 2.1.50

Correct option is (D).


Given Boolean expression is
AD + ABCD + ACD + AB + AC D + A B
On minimization using Boolean algebra, we get
AD + ABCD + ACD + AB + AC D + A B
= AD _1 + BC + C + C i + AB + A B

= AD + AB + A B

= AD + A _B + B i = AD + A
= _A + A i_D + A i = A + D

in
.
o

NOTE :

Here, it must be noted that, we got first the expression ^AD + A h , and this is also
given in option (B). At first glance it seems to be Answer but it is not. It can be
minimized further into ^A + D h .

SOL 2.1.51

c
.
ia

Correct option is (C).


We redraw the given digital circuit as

d
o

n
.
w
w

Output Y is given by

Y = AB + AB = A 5 B = A _XOR i B

SOL 2.1.52

Correct option is (A).


We have the switching circuit as

Buy Online: shop.nodia.co.in


*Shipping Free*

*Maximum Discount*

GATE STUDY PACKAGE

Electronics & Communication

Sample Chapter of

Digital Electronics

(Vol-6, GATE Study Package)

Now, we consider the parallel switches as shown below.

Page 135
Chap 2
Boolean Algebra and
Logic Simplification

The logic expression for parallel switches is obtained as


L = A+B
Again, we consider the series connected switches

i. n

o
c
.
a

i
d

o
n

The logic expression for series connected switches is

.
w
w

L = A:B
Hence, the logic expression for given circuit is

SOL 2.1.53

L = _A + B i : _C + D i : E

Correct option is (A).


We have to check the correctness of each options.
Option (A)
On minimizing L.H.S. of the equation, we have
or

X + XY = _X + X i_X + Y i = X + Y ! X
X + XY ! X

Option (B)
On minimizing L.H.S. of the equation, we have
X _X + Y i = XX + XY = XY
X _X + Y i = XY

or
Option (C)
On minimizing L.H.S. of the equation, we have
X + XY = X _1 + Y i = X
X + XY = X

or
Option (D)
On minimizing L.H.S. of the equation, we have
or

ZX + ZXY = Z _X + XY i = Z _X + Y i = ZX + ZY
ZX + ZXY = ZX + ZY

Buy Online: shop.nodia.co.in


*Shipping Free*

*Maximum Discount*

GATE STUDY PACKAGE


Electronics & Communication
10 Subject-wise books by R. K. Kanodia
General Aptitude
Engineering Mathematics
Networks
Electronic Devices
Analog Electronics
Digital Electronics
Signals & Systems
Control Systems Communication Systems
Electromagnetics

Page 136
Chap 2

SOL 2.1.54

Correct option is (D).


We have the switching circuit diagram as

Boolean Algebra and


Logic Simplification

in
.
o

From the circuit diagram, we deduce that LED will glow when
Z = Low _ 0 i , and Z will be low only when X or Y or both will be
High. Now, we consider the different input conditions as
1. For A = B = 1,

c
.
ia

X = 1 and Y = 0 & Z = 0 ; LED glows


2.

For A = B = 0 ,

3.

X = 0 and Y = 1 & Z = 0 ; LED glows


For A = 0 , B = 1;

d
o

n
.
w
w

4.

X = 0 and Y = 0 & Z = 1; LED doesnt glow


For A = 1, B = 0 ;
X = 0 and Y = 0 ( Z = 1; LED doesnt glow

SOL 2.1.55

Correct option is (C).


We have the Boolean identity

AB + AC = _A + C i_A + B i
Dual form of any identity can be found by replacing all AND function
to OR and vice-versa. Hence, dual form of the expression is given as
_A + B i : _A + C i = _A : C i + _A : B i

SOL 2.1.56

Correct option is (D).


Dual form of any identity can be found by replacing all AND functions
to OR functions and vice-versa. Now, we have the Boolean expression
as
A + 7B + _AC iA + D
So, the dual form of the expression is given as
or

A : 7B : _A + C iA : D
A 7B _A + C iA : D

Buy Online: shop.nodia.co.in


*Shipping Free*

*Maximum Discount*

GATE STUDY PACKAGE

Sample Chapter of

SOL 2.1.57

Electronics & Communication

Digital Electronics

(Vol-6, GATE Study Package)

Correct option is (B).


We have the Boolean expression as

Page 137
Chap 2
Boolean Algebra and
Logic Simplification

Z = x 5 y 5 xy
On minimizing the expression, we have

Z = x 5 7y 5 xyA
= x 5 7yxy + yxyA
= x 5 7y _x + y i + 0A
= x 5 7yx + 0A
= x : yx + x : yx
= x _y + x i + xy = x + xy + xy
= x _1 + y i + xy = x + xy

i. n

= x+y

SOL 2.1.58

Correct option is (C).


We have the Boolean expression as

o
c
.
a

i
d

_X + W i_Y 5 Z i + XW l
On simplification and minimization of the Boolean expression, we get
_X + W i_Y 5 Z i + XW l = _X + W i_YZ l + Y l Z i + XW l
= XYZ l + XY l Z + WYZ l + WY l Z + XW l
From consensus theorem, we have

o
n

.
w
w

AB + AC + BC = AB + AC
So, eliminating the redundant term in the expression, we get

i.e. the minimized expression


WY l Z + XW l + WYZ l

SOL 2.1.59

of

_X + W i_Y 5 Z i + XW l

is

Correct option is (B).


We have the Boolean expression

F = _x + y i_x + y li
On simplification, we get the expression
Fsimplified = x + xy l + xy + yy l

...(1)

= x _1 + y l + y i
...(2)
=x
Hence, the logic diagram for the expression (1) and (2) is shown below.

Buy Online: shop.nodia.co.in


*Shipping Free*

*Maximum Discount*

GATE STUDY PACKAGE


Electronics & Communication
10 Subject-wise books by R. K. Kanodia
General Aptitude
Engineering Mathematics
Networks
Electronic Devices
Analog Electronics
Digital Electronics
Signals & Systems
Control Systems Communication Systems
Electromagnetics

Page 138
Chap 2
Boolean Algebra and
Logic Simplification

SOL 2.1.60

Correct option is (C).


To get output at High state, i.e. to stop the Machine operation; either
P is low (no paper in paper feeder) or Q and R are high (jam in paper
path). Hence, logic expression for output S is

in
.
o

S = P+Q:R
So, the logic circuit for the given condition is drawn as

c
.
ia

d
o

SOL 2.1.61

n
.
w
w

Correct option is (B).


We redraw the given logic circuit as

DRIVE is active-HIGH, and it will go high only when


X =Y=0
X will be LOW only when either A8 and A9 is HIGH.

Buy Online: shop.nodia.co.in


*Shipping Free*

*Maximum Discount*

GATE STUDY PACKAGE

Sample Chapter of

Electronics & Communication

Digital Electronics

(Vol-6, GATE Study Package)

Y will be LOW only when W = 0 and A7 = 0


W will be LOW only when A0 through A6 are all HIGH.
Putting this all together, we have the condition for DRIVE to be high
as

Boolean Algebra and


Logic Simplification

A0 = A1 = A2 = A3 = A4 = A5 = A6 = 1
A7 = 0
A8 or A9 or both are 1

and either

SOL 2.1.62

Page 139
Chap 2

Correct option is (A).


We have the wired-OR logic circuit as shown below.

i. n

o
c
.
a

o
n

i
d

For the logic circuit, the output is

SOL 2.1.63

.
w
w

Y = _A + B i + _C + D i

Correct option is (C).


We have the two level AND-OR gate as shown below.

Now, all the gates are replaced by NOR gate. So, we get the modified
circuit as

Hence, the output of the modified network is


Z = _A + B i + _C + D i
= _A + B i : _C + D i
= dual of F = F D

Buy Online: shop.nodia.co.in


*Shipping Free*

*Maximum Discount*

GATE STUDY PACKAGE


Electronics & Communication
10 Subject-wise books by R. K. Kanodia
General Aptitude
Engineering Mathematics
Networks
Electronic Devices
Analog Electronics
Digital Electronics
Signals & Systems
Control Systems Communication Systems
Electromagnetics

Page 140
Chap 2

SOL 2.1.64

Boolean Algebra and


Logic Simplification

Correct option is (C).


The output function for the given circuit is
f = xyz + wyz + wxz
Let
y = A; x = B ; w = C
then
f = Z _AB + AC + BC i
Using consensus theorem, we conclude that BC term is redundant. So,
we have
f = Z _AB + AC i
or
f = xyz + wyz
Hence, gate 3 is redundant.

SOL 2.1.65

Correct option is (B).


We have the logic circuit as shown below

in
.
o

c
.
ia

d
o

n
.
w
w

From the Boolean algebra, we have

By using the above conversion, we redraw the given logic circuit as

Hence, the output of the logic circuit is


Y = P+Q+R+S

Buy Online: shop.nodia.co.in


*Shipping Free*

*Maximum Discount*

GATE STUDY PACKAGE

Electronics & Communication

Sample Chapter of

SOL 2.1.66

Digital Electronics

(Vol-6, GATE Study Package)

Correct option is (A).


All the given circuit in the options include NOR gate. So, we implement
the given expression using NOR gate as

Page 141
Chap 2
Boolean Algebra and
Logic Simplification

X = AB + CD = (A + B ) (C + D )
= (A + B ) + (C + D )

i. n

SOL 2.1.67

i
d

o
n

.
w
w

o
c
.
a

Correct option is (B).


In order to convert the given circuit into all NOR, we apply the bubbles
at the input terminals of gates as shown below.

From the Boolean algebra, we have

and

Therefore, by using the above conversion, we get the logic circuit with
NOR gates.

Buy Online: shop.nodia.co.in


*Shipping Free*

*Maximum Discount*

GATE STUDY PACKAGE


Electronics & Communication
10 Subject-wise books by R. K. Kanodia
General Aptitude
Engineering Mathematics
Networks
Electronic Devices
Analog Electronics
Digital Electronics
Signals & Systems
Control Systems Communication Systems
Electromagnetics

Page 142
Chap 2

SOL 2.1.68

Boolean Algebra and


Logic Simplification

Correct option is (D).


In order to convert the given circuit using only NAND gate, we apply
bubbles at the input terminal of each gates as shown below.

From the Boolean algebra, we have

and

in
.
o

c
.
ia

Therefore, by using the above conversion, we get the logic circuit with
NAND gates as

d
o

n
.
w
w

w
SOL 2.1.69

Correct option is (A).


Figure I
We have the combinational circuit in Figure - I as

So, the output of the circuit is


F1 = X + (X + Y ) = X (X + Y )
= X X + XY = XY
Figure II
We have the combinational circuit in Figure - II as

Buy Online: shop.nodia.co.in


*Shipping Free*

*Maximum Discount*

GATE STUDY PACKAGE

Sample Chapter of

Electronics & Communication

Digital Electronics

(Vol-6, GATE Study Package)

Page 143
Chap 2
Boolean Algebra and
Logic Simplification

So, the output of the circuit is


F2 = X $ Y
Hence, we have
F1 ! F2
i.e. the output of the given two circuits are entirely different.

***********

i. n

i
d

o
n

.
w
w

o
c
.
a

Buy Online: shop.nodia.co.in


*Shipping Free*

*Maximum Discount*

GATE STUDY PACKAGE


Electronics & Communication
10 Subject-wise books by R. K. Kanodia
General Aptitude
Engineering Mathematics
Networks
Electronic Devices
Analog Electronics
Digital Electronics
Signals & Systems
Control Systems Communication Systems
Electromagnetics

SOLUTIONS 2.2

Page 144
Chap 2
Boolean Algebra and
Logic Simplification

SOL 2.2.1

Correct answer is 0.
Given Boolean function is

F = A + AB + ABC = A _1 + B + BC i = A
Therefore, no gate is required to implement this function.

SOL 2.2.2

Correct answer is 3.
As the given expression is to be realized using one type of 2-input gates.
So, we may use universal gates (NAND, NOR) for realization. Now, we
implement the given function using NAND and NOR gates.
1. NAND Implementation: For NAND implementation, we rewrite the
given expression

in
.
o

c
.
ia

Y = A + BC = A + BC = _A i : _BC i
So, the logic circuit can be implemented as

d
o

n
.
w
w

2. NOR Implementation: For NOR implementation, we rewrite the


given expression as
[Distributive
Y = A + BC = _A + B i_A + C i ;
property]
or

Y = _A + B i_A + C i
= _A + B i + _A + C i

Thus, to implement given circuit, minimum 3 gates are required.

Buy Online: shop.nodia.co.in


*Shipping Free*

*Maximum Discount*

GATE STUDY PACKAGE

Electronics & Communication

Sample Chapter of

SOL 2.2.3

SOL 2.2.4

Digital Electronics

(Vol-6, GATE Study Package)

Correct answer is 31.


We know that, if any one input of the OR gate becomes high logic (1),
OR gate gives the high logic (1) output. The output is High only for
the case when all the inputs are at Low logic (0). Now, for 5-inputs,
we have
Total number of input conditions = 2 5 = 32
Out of the 32 conditions, all inputs are zero (0) for only one condition.
i.e. for only one condition the output is low.
Hence, 31 input conditions produce the high output from a five-input
OR gate.

Page 145
Chap 2
Boolean Algebra and
Logic Simplification

i. n

Correct answer is 2.
The Boolean function of two variables x and y are defined as
f _0, 0i = f _0, 1i = f _1, 1i = 1 and f _1, 0i = 0
For the Boolean function, we obtain the truth table as
x

i
d

o
n

.
w
w

o
c
.
a

0
1

From the truth table, we define the function f as


f = xy
So,
f = xy = x + y
Hence, the function _ f i can be implemented using 2 input NOR and
2-input OR gate as shown below.

Thus, the total cost for the logic circuit will be 2 units.

SOL 2.2.5

Correct answer is 6.
The circuit is as follows

Buy Online: shop.nodia.co.in


*Shipping Free*

*Maximum Discount*

GATE STUDY PACKAGE


Electronics & Communication
10 Subject-wise books by R. K. Kanodia
General Aptitude
Engineering Mathematics
Networks
Electronic Devices
Analog Electronics
Digital Electronics
Signals & Systems
Control Systems Communication Systems
Electromagnetics

Page 146
Chap 2

SOL 2.2.6

Boolean Algebra and


Logic Simplification

Correct answer is 011.


For given logic circuit, output X = 1, if all inputs to AND gate are
High. The output for the circuit can be expressed as
X = _A 5 B i : _B 9 C i : C
Also, we have
A 5 B = 1 if A ! B
and
B 9 C = 1 if B = C
Hence, for X = 1, the required conditions are
C must be High
B =C=1
A =0

SOL 2.2.7

(since A ! B )

Correct answer is 0.
Given the logic equation,

in
.
o

7X + Z #Y + _Z + XY i-A#X + Z _X + Y i- = 1
and
X =1
So, by substituting X = 1 and X = 0 in the logic equation, we get

c
.
ia

71 + Z #Y + _Z + 1Y i-A : 80 + Z _1 + Y iB = 1
71A8Z _ 1 iB = 1

or
Hence, we have

SOL 2.2.8

Z = 1 or Z = 0

d
o

n
.
w
w

Correct answer is 0.
Given logic expression is
A _A + B i_A + B + C i
On solving the expression, we have

A _A + B i_A + B + C i = _AA + AB i_A + B + C i


= _A + AB i_A + B + C i
= A + AB + A : AB + A : B : B + AC + ABC
= A + AB + AC + ABC
= A _1 + B + C + BC i = A
Therefore, no gate is required to implement this function.

SOL 2.2.9

Correct answer is 1.
From the given circuit, we can observe that input to last XNOR gate is
same. So, the XNOR output is given by (Let input is X )
Z = X:X+X:X = X+X = 1
i.e. the output will be High (logic 1), irrespective of the inputs A and
B.

Buy Online: shop.nodia.co.in


*Shipping Free*

*Maximum Discount*

GATE STUDY PACKAGE

Sample Chapter of

SOL 2.2.10

Electronics & Communication

Digital Electronics

(Vol-6, GATE Study Package)

Correct answer is 65536.


n
The number of distinct Boolean expressions of n variables is 2 2 . Since,
we have
n =4
Hence, the number of distinct Boolean expressions is

Page 147
Chap 2
Boolean Algebra and
Logic Simplification

2 2 = 2 2 = 2 16 = 65536
n

SOL 2.2.11

Correct answer is 256.


The number of duals of distinct boolean expressions of n variables
is 2 2 . Since, we have n = 4 . Hence, the number of duals of distinct
Boolean expressions is
n-1

22

SOL 2.2.12

n-1

= 2 2 = 2 2 = 2 8 = 256
4-1

i. n

o
c
.
a

Correct answer is 3.
To implement the given function using NAND and NOR gates, we
rewrite the given function as

i
d

o
n

Y = ABCD = ABCD = AB + CD
So, the equivalent circuit for the Boolean function is

.
w
w

Therefore, two NAND gates and one NOR gate is required to implement
the function Y = ABCD .

***********

Buy Online: shop.nodia.co.in


*Shipping Free*

*Maximum Discount*

GATE STUDY PACKAGE


Electronics & Communication
10 Subject-wise books by R. K. Kanodia
General Aptitude
Engineering Mathematics
Networks
Electronic Devices
Analog Electronics
Digital Electronics
Signals & Systems
Control Systems Communication Systems
Electromagnetics

SOLUTIONS 2.3

Page 148
Chap 2
Boolean Algebra and
Logic Simplification

SOL 2.3.1

Correct option is (A).


NAND gate output Y = A : B
Where, A and B are inputs, if inputs are connected together, i.e. A = B
Y = A:A = A

SOL 2.3.2

Correct option is (C).

SOL 2.3.3

Correct option is (A).

SOL 2.3.4

Correct option is (D).

SOL 2.3.5

Correct option is (A).

SOL 2.3.6

in
.
o

c
.
ia

d
o

n
.
w
w

Correct option is (B).

SOL 2.3.7

Correct option is (A).

SOL 2.3.8

Correct option is (A).


AND gate output Y = A : B : C
If any one of the inputs is LOW then output becomes LOW. Output
will HIGH only when all the inputs are HIGH.
For 3-inputs, 8 input possibilities are there, out of which only one case
has all inputs high.

SOL 2.3.9

Correct option is (C).


OR gate output
Y = A+B+C
If any one input is HIGH, output will be HIGH. Output will be LOW

Buy Online: shop.nodia.co.in


*Shipping Free*

*Maximum Discount*

GATE STUDY PACKAGE

Sample Chapter of

Electronics & Communication

Digital Electronics

(Vol-6, GATE Study Package)

only when all three inputs become LOW.


Out of 8 input possibilities, seven cases have one or more inputs high
results in HIGH.

SOL 2.3.10

Correct option is (A).

SOL 2.3.11

Correct option is (C).

SOL 2.3.12

Correct option is (A).

SOL 2.3.13

Correct option is (C)

SOL 2.3.14

Correct option is (B).

SOL 2.3.15

Correct option is (B).

o
c
.
a

i
d

Output of two input _A, B i NOR gate,


Using Demorgan theorem,
AND gate

Boolean Algebra and


Logic Simplification

i. n

o
n

.
w
w

Page 149
Chap 2

Y = A+B
Y = A:B =

negative-

SOL 2.3.16

Correct option is (C).

SOL 2.3.17

Correct option is (A).

SOL 2.3.18

Correct option is (D).


Two-input NAND gate using two-input NOR gate is realized
as :

Buy Online: shop.nodia.co.in


*Shipping Free*

*Maximum Discount*

GATE STUDY PACKAGE


Electronics & Communication
10 Subject-wise books by R. K. Kanodia
General Aptitude
Engineering Mathematics
Networks
Electronic Devices
Analog Electronics
Digital Electronics
Signals & Systems
Control Systems Communication Systems
Electromagnetics

Page 150
Chap 2

SOL 2.3.19

Correct option is (A).

SOL 2.3.20

Correct option is (C).

SOL 2.3.21

Correct option is (A).


The Ex-NOR gate output is HIGH only when both input bits of it are
equal.
Hence, Ex-NOR gate is suitable for bit comparison.

SOL 2.3.22

Correct option is (C).

SOL 2.3.23

Correct option is (B).

SOL 2.3.24

Correct option is (C).


AND : Y = A : B = B : A
OR : Y = A + B = B + A

Boolean Algebra and


Logic Simplification

in
.
o

c
.
ia

d
o

SOL 2.3.25

SOL 2.3.26

n
.
w
w

Correct option is (A).

Correct option is (D).


The truth table for Ex-NOR gate is,
Inputs

Output

Y = A9B

The output is 1 for like inputs and 0 for unlike inputs.

SOL 2.3.27

Correct option is (C).


The truth table for NAND and OR is,

Buy Online: shop.nodia.co.in


*Shipping Free*

*Maximum Discount*

GATE STUDY PACKAGE

Electronics & Communication

Sample Chapter of

Digital Electronics

Inputs

(Vol-6, GATE Study Package)

Page 151
Chap 2

Outputs

X = A:B

Y = A+B

Boolean Algebra and


Logic Simplification

For both NAND and OR gates, all output entries are logic 1 except
for one entry.

i. n

SOL 2.3.28

Correct option is (A).

SOL 2.3.29

Correct option is (A).

SOL 2.3.30

Correct option is (C).


Taking of two times complement results the original function.

i
d

o
n

.
w
w

o
c
.
a

Y = AB + AB

Complement of Y = Y = AB + AB

SOL 2.3.31

Complement of Y = Y = Y = AB + AB = AB + AB

Correct option is (D).

SOL 2.3.32

Correct option is (D).

SOL 2.3.33

Correct option is (A).

SOL 2.3.34

Correct option is (C).

SOL 2.3.35

Correct option is (B).

SOL 2.3.36

Correct option is (C).


Y = A + B = A $ B = NAND gate logic

Buy Online: shop.nodia.co.in


*Shipping Free*

*Maximum Discount*

GATE STUDY PACKAGE


Electronics & Communication
10 Subject-wise books by R. K. Kanodia
General Aptitude
Engineering Mathematics
Networks
Electronic Devices
Analog Electronics
Digital Electronics
Signals & Systems
Control Systems Communication Systems
Electromagnetics

Page 152
Chap 2

SOL 2.3.37

Correct option is (D).


Y = A.B = A + B = NOR gate logic

Boolean Algebra and


Logic Simplification
SOL 2.3.38

Correct option is (B).


Y = A.B = A + B = A + B = OR gate logic

SOL 2.3.39

Correct option is (A).


Y = A + B = A $ B = AND gate logic

SOL 2.3.40

Correct option is (D).

SOL 2.3.41

Correct option is (D).

SOL 2.3.42

Correct option is (D).

SOL 2.3.43

Correct option is (B).

SOL 2.3.44

Correct option is (C).

in
.
o

c
.
ia

d
o

n
.
w
w

SOL 2.3.45

Correct option is (D).

SOL 2.3.46

Correct option is (D).


NAND and NOR gates are universal gates. Any logic function can be
realized using only NAND or only NOR gates.
So, AND, OR and NOT gates are not necessary to realize any logic
function.

SOL 2.3.47

Correct option is (B).


X = A.B = A + B = NOR gate logic
Hence, output will be HIGH if and only if both the inputs are low.

Buy Online: shop.nodia.co.in


*Shipping Free*

*Maximum Discount*

GATE STUDY PACKAGE

Sample Chapter of

SOL 2.3.48

Electronics & Communication

Digital Electronics

(Vol-6, GATE Study Package)

Correct option is (B).

Page 153
Chap 2
Boolean Algebra and
Logic Simplification

SOL 2.3.49

Correct option is (A).


When the no. of inputs in the XOR gate is even then output is 0 and
when the no. of inputs is odd then output is 1.

SOL 2.3.50

Correct option is (B).


NAND and NOR gates do not follow the associative property.
Now check for NOR gate, (A + B ) + C is equal to or not equal to
A + (B + C ) .

SOL 2.3.51

Correct option is (B).

i. n

o
c
.
a

(A + B ) + C ! A + (B + C )
(A + B ) + C ! A + (B + C )

or
or

o
n

i
d

A + AB + ABC + ABCD + ABCDE + ....


= A (1 + B + BC + BCD + BCDE + ...)

SOL 2.3.52

.
w
w

= A (1 + X) = A

Correct option is (A).


A + AB + A BC + A B C D + ... = A + A (B + BC + BC D + ...)
= A + A (X ) = (A + A) (A + X )
= A+X
= A + B + BC + B C D + ... and so on.
= A + B + C + ...

SOL 2.3.53

Correct option is (D).


The number of combinations for five inputs is = 2 5 = 32 .

SOL 2.3.54

Correct option is (B).

SOL 2.3.55

Correct option is (C).

SOL 2.3.56

Correct option is (D).

Buy Online: shop.nodia.co.in


*Shipping Free*

*Maximum Discount*

GATE STUDY PACKAGE


Electronics & Communication
10 Subject-wise books by R. K. Kanodia
General Aptitude
Engineering Mathematics
Networks
Electronic Devices
Analog Electronics
Digital Electronics
Signals & Systems
Control Systems Communication Systems
Electromagnetics

Page 154
Chap 2

SOL 2.3.57

Boolean Algebra and


Logic Simplification

Correct option is (C).


Output of each option is
(A) Y = 0.1 = 0

(B) Y = 0 + 1 = 1 = 0

(C) Y = 0 $ 1 = 0 = 1

(D) Y = 0 9 1 = 0

SOL 2.3.58

Correct option is (B).

SOL 2.3.59

Correct option is (C).

SOL 2.3.60

Correct option is (A).

in
.
o

***********

c
.
ia

d
o

n
.
w
w

Buy Online: shop.nodia.co.in


*Shipping Free*

*Maximum Discount*

You might also like